MBE Torts

Ace your homework & exams now with Quizwiz!

A trainer of homing pigeons brought several of them to a park that he often used for training. He had trained this group of pigeons carefully and was confident that they would readily find their way home. When they were released, one of the pigeons inexplicably turned in the opposite direction from home. Several blocks away at the other end of the park, it collided with a radio-controlled model airplane that its owner had just purchased and was trying out for the first time. The collision sent the airplane out of control; it dipped low across a highway and was struck and run over by a truck. The airplane owner sued the pigeon trainer for the destruction of his airplane. The parties stipulated to the above facts and the airplane owner presented evidence of his damages. The trainer then moved for a directed verdict. Should it be granted? A No, because the trainer's pigeon caused the destruction of the airplane. B No, because the jury could find negligence on the trainer's part under the doctrine of res ipsa loquitur. C Yes, because the truck, rather than the pigeon, was the direct cause of the airplane's destruction. D Yes, because the trainer took reasonable care in training his pigeons.

he court should grant a directed verdict for the trainer because the airplane owner has not shown that the trainer breached any duty that he owed to him. A prima facie case of negligence requires plaintiff to show the following elements: (i) the existence of a duty on the part of the defendant to conform to a specific standard of conduct for the protection of the plaintiff against unreasonable risk of injury, (ii) breach of that duty by the defendant, (iii) that the breach of duty was the actual and proximate cause of plaintiff's injury, and (iv) damage to plaintiff's person or property. Here, it is doubtful that the trainer's releasing his pigeons created any duty to other users of the park. To the extent that it did, the fact that he had taken great care to train them to return directly to their roosts indicates that he did not breach his duty to the airplane owner. Because the airplane owner has offered no other evidence of negligence, nor any reason to impose strict liability on the trainer (as discussed below), the trainer's motion for a directed verdict should be granted. (A) is incorrect because that choice indicates the imposition of a strict liability standard on the trainer. The owner of a domestic or inherently nondangerous animal is not strictly liable for the injuries it causes. The conduct of the trainer's homing pigeon would not make the trainer liable in the absence of some negligence on his part. (B) is incorrect because the doctrine of res ipsa loquitur applies only to situations where the fact that a particular injury occurred itself establishes that defendant breached a duty. If the doctrine is applicable, no directed verdict may be given for defendant because plaintiff has established a prima facie case. However, the accident must be the type that would not normally occur unless someone was negligent. The collision between the trainer's homing pigeon and the model airplane is not that type of accident; by itself, it provides no suggestion that anyone was negligent. (C) is incorrect because the truck is not a superseding force that breaks "the causal connection" between the action of the trainer's pigeon and the airplane's destruction. In indirect cause cases, where a force came into motion after defendant's act and combined with it to cause injury to plaintiff, defendant will still be potentially liable for foreseeable intervening forces that are within the increased risk caused by his acts. Even if the intervening force is independent (i.e., not a natural response or reaction to the situation), it will be foreseeable where defendant's negligence increased the risk that the independent force would cause harm. Hence, if the trainer were negligent in releasing his pigeon, the fact that the destruction of the airplane was directly caused by the truck would not relieve the trainer from liability, because the initial collision with the pigeon caused the airplane to go out of control and created a substantial risk that it would be damaged by an intervening force.

Which of the following is true of the duty owed to a licensee by a landowner? A The landowner must inspect for dangerous conditions on the land. B The landowner owes a duty to warn of or make safe known dangerous conditions on the land of which the licensee is not aware. C The landowner owes no duty to protect the licensee from active operations on the land. D The landowner must repair known dangerous conditions on the land of which the licensee is not aware.

A landowner owes a duty to a licensee to warn of or make safe known dangerous conditions on the land of which the licensee is not aware. A licensee is one who enters on land with the landowner's permission, express or implied, for her own purpose or business, rather than for the landowner's benefit. The owner has a duty to warn of or make safe a dangerous condition known to the owner that creates an unreasonable risk of harm to the licensee and that the licensee is unlikely to discover. As to a licensee, the landowner does not need to inspect for dangerous conditions on the land. Similarly, the landowner need not repair known dangerous conditions on the land of which the licensee is not aware; a warning generally will suffice. The landowner does have a duty to protect the licensee from active operations on the land. The owner has a duty to exercise reasonable care in the conduct of active operations for the protection of a licensee whom he knows to be on the property.

For purposes of private nuisance, the interference with the plaintiff's use of the land is unreasonable only if: A The interference is offensive or annoying to an average person in the community. B The remedy of damages is unavailable or inadequate. C The nuisance is a "nuisance per se." D The severity of the plaintiff's inflicted injury outweighs the utility of the defendant's conduct.

The interference with the plaintiff's use of the land will be considered unreasonable under nuisance law when the severity of the plaintiff's inflicted injury outweighs the utility of the defendant's conduct. In balancing these respective interests, courts take into account that every person is entitled to use his own land in a reasonable way, considering the neighborhood, land values, and existence of any alternative courses of conduct open to the defendant. Whether the interference is offensive or annoying to an average person in the community is the test for whether the interference is substantial, which is a separate requirement for establishing a nuisance. Whether the remedy of damages is unavailable or inadequate determines only whether the plaintiff may be able to obtain an injunction remedy. A nuisance is sometimes called a "nuisance per se" when it is based on strict liability (e.g., a nuisance arising from an abnormally dangerous activity).

In contrast to products liability cases based on negligence, those based on strict liability do not: A Require an injured bystander to be foreseeable. B Require that suppliers have an opportunity to inspect. C Prohibit recovery of solely economic losses. D Impose liability when an intermediary negligently failed to discover the defect.

Unlike with products liability cases based on negligence, those based on strict liability do not require that suppliers have an opportunity to inspect. Thus, for a case based on the sale of a defective product, a retailer in a strict liability action may be liable for a manufacturing or design defect simply for being a commercial supplier of that defective product, even if it had no opportunity to inspect the manufacturer's product before selling it. In a negligence action, the supplier's negligence must be proved. Products liability cases based on negligence and those based on strict liability both require that an injured bystander be foreseeable. While privity is not required in these cases, and bystanders are protected and may bring a claim under either theory, they must be foreseeable plaintiffs. Liability under these theories applies only to foreseeable plaintiffs. Products liability cases based on negligence and those based on strict liability both prohibit recovery of solely economic losses. The types of damages recoverable under both theories are the same: personal injury and property damages. Economic loss cannot be the sole damage claim. As under claims based on negligence, those based on strict liability will impose liability even though an intermediary negligently failed to discover the defect. The same concepts of proximate cause govern negligence and strict liability actions. The negligent failure of an intermediary to discover a defect is not a superseding cause and does not cut off the supplier's strict liability. However, if the intermediary's conduct becomes something more than ordinary foreseeable negligence, then it does become a superseding cause.

How is "actual malice" defined for purposes of the constitutional law of defamation? A Knowledge that the statement was false, or reckless disregard by the defendant as to the statement's truth or falsity. B At least negligence by the defendant as to the statement's truth or falsity. C Ill will or spite towards the plaintiff by the defendant. D Knowledge that the statement will defame the plaintiff, or reckless disregard by the defendant as to the effect of the defamatory language.

"Actual malice," which must be established by a public official or public figure to recover for defamation, was defined by the Supreme Court as (i) knowledge that the statement was false, or (ii) reckless disregard by the defendant as to the statement's truth or falsity. Negligence by the defendant as to the statement's truth or falsity is the minimum standard imposed by the Court on a private figure when the defamation involves a matter of public concern; actual malice is a higher standard. Ill will or spite towards the plaintiff by the defendant is how malice was defined at common law, but is not used by the Supreme Court to define malice. Knowledge that the statement will defame the plaintiff or reckless disregard by the defendant as to the effect of the defamatory language is not how malice was defined by the Supreme Court. The Court used actual malice to describe the defendant's necessary state of mind as to the statement's truth or falsity, not its defamatory nature.

Which of the following intentional conduct by the defendant is LEAST likely to constitute a trespass to land? A Exploding a mine on his own land which causes concussion damage to the plaintiff's land. B Throwing a rock onto a plaintiff's driveway which causes no damage. C Flooding his own land which causes water damage to the plaintiff's land from the overflow. D Chasing someone from his own land onto the plaintiff's land which causes no damage.

(A) If a defendant explodes a mine on his land which causes only concussion damage to a plaintiff's land, this will likely not constitute a trespass to land. A trespass to land requires a physical invasion of a plaintiff's real property by a defendant. If no physical object enters onto the plaintiff's land due to the defendant's actions, courts will generally not treat the defendant's conduct as a trespass to land. Instead, this will constitute a case of nuisance or strict liability (if ultrahazardous activities are involved). If a defendant throws a rock onto the plaintiff's driveway, causes water to flow onto the plaintiff's land, or chases someone onto the plaintiff's land, this will likely constitute a trespass to land. Although in none of these scenarios does the defendant actually enter upon the plaintiff's land, a trespass to land does not require that the defendant personally come onto the land. The tort of trespass to land protects a plaintiff's exclusive possession of realty from physical invasion, and all that is required to satisfy this element is a physical invasion of the plaintiff's land. Furthermore, damages are not required for this tort. The defendant has committed a trespass even if the property was not damaged.

The "shopkeeper's privilege" allows a shopkeeper to avoid liability for false imprisonment when detaining a suspect that he reasonably believes has committed a theft. The shopkeeper also must: A Conduct the detention in a reasonable manner and detain the suspect for only a reasonable time. B Conduct the detention in a reasonable manner and notify the police in a reasonable amount of time. C Detain the suspect for only a reasonable time and notify the police in a reasonable amount of time. D Conduct the detention in a reasonable manner, detain the suspect for only a reasonable time, and notify the police in a reasonable amount of time.

(A) In addition to having a reasonable belief as to the fact of theft, a shopkeeper is required to conduct the detention in a reasonable manner and detain the suspect for a reasonable period of time for the privilege to apply. By statute in some states and case law in others, shopkeepers have been given a privilege to detain someone suspected of shoplifting and thus avoid liability for false imprisonment. The following conditions must be satisfied: (i) there must be a reasonable belief as to the fact of theft; (ii) the detention must be conducted in a reasonable manner and only nondeadly force can be used; and (iii) the detention must be only for a reasonable period of time and only for the purpose of making an investigation. A shopkeeper is not required to notify the police in a reasonable amount of time to avoid liability for false imprisonment when detaining a suspect for shoplifting.

An inexperienced worker who was instructed to clean the floors of a store mixed ammonia and chlorine bleach in a large pail. Both he and a customer who was standing nearby were overcome by fumes and suffered lung damage. The customer sued the worker, alleging negli- gence. In defense, the worker presented uncon- troverted evidence that he could not read the warning labels on the containers and that, while he knew he was mixing ammonia and bleach, he had never been made aware of the danger of mixing the two chemicals. Nevertheless, the jury found him liable for the customer's injuries. If the worker challenges the verdict on appeal, how should the appellate court rule? (A) Uphold the verdict, because it was a deter- mination that a reasonable person should have known of the danger. (B) Uphold the verdict, because it was a deter- mination that the worker's evidence was not believed. (C) Overrule the verdict, because it is inconsis- tent with the evidence. (D) Overruletheverdict,becausetheworker's lack of knowledge of the danger should have been taken into account.

(A) The appellate court should uphold the verdict because it is a determination by the jury that the worker did not act as a reasonable person would have acted. In a negligence action, the defendant's conduct is measured against the reasonable person, an objective standard. The defendant must act as would a person of average mental ability, and he is deemed to have knowledge of things known by an average member of the community; individual shortcomings or ignorance of that particular defendant are not considered. Here, the verdict of negligence was a determination by the jury that a reasonable person should have known of the danger, regardless of the fact that the defendant did not. Hence, the verdict should be upheld because there is no basis in the facts for overturning it. (B) is not as good a choice as (A) because nothing suggests that the jury did not believe the worker's evidence. The worker admittedly knew that he was mixing ammonia and bleach. Given that he was also injured by the fumes, his assertion that he was not aware of the danger was very believable. (C) is incorrect because the appellate court will not overrule a determination of negli- gence by the trier of fact unless no reasonable jury could have made that determination. Given that the standard of care requires the exercise of knowledge and ability of the average person, a reasonable jury could have found negligence here. (D) is incorrect because, as discussed above, the worker's lack of knowledge is not taken into account when determining the standard of care for negligence.

A pilot was flying her small airplane when she realized that she was rapidly losing fuel and would not make it to the nearest airport. Looking down, she could find no large open space on which to attempt a landing except for a highway off to her left and a nearby lake about a mile to her right. She considered ditching the plane in the water but decided against it under the circumstances. As the pilot maneuvered over the highway and saw a long section free of any overpass or obstruction, her engine sputtered and died. In a barely controlled glide, the pilot descended onto the highway, but her left wing sideswiped the median and her plane veered to the right, crashing into a car. The plane and car catapulted into a fence, severely injuring both the pilot and driver. The driver brought an action for personal injuries against the pilot. At trial, the above facts were established, and the parties stipulated that the sudden loss of fuel was due to a defect in the fuel system that could not have been discovered by the pilot. At the close of the evidence, both parties moved for a directed verdict. How should the court rule? (A) Deny both motions, because the jury could decide that the pilot's selection of the high- way rather than the lake was not a reason- able choice under the circumstances. (B) Grant the driver's motion, because the evidence establishes that his injuries were the result of a defect in the pilot's plane. (C) Grant the pilot's motion, because the parties stipulated that she was not negligent in failing to discover the defect in her fuel system. (D) Grant the pilot's motion, because she made the decision to land on the highway rather than the lake under emergency conditions.

(A) The court should deny both motions and submit the case to the jury. Through process of elimina- tion, this has to be a negligence action. Clearly the pilot did not commit an intentional tort, and the driver cannot sue the pilot, the plane's owner, in strict liability. The jury could determine that the pilot was negligent in selection of the landing site; hence, the court should deny the pilot's motion. The court should also deny the driver's motion because this is not a strict liability action; negligent conduct needs to be established. Hence, (B) is incorrect. (C) is incorrect because even though the pilot was not negligent in discovering the defect, she could have been negligent in selecting the landing site. (D) is incorrect because in an emergency, the pilot is held to the standard of care of a reasonable person in an emergency. It is up to the jury to determine whether she acted reasonably under the emergency circumstances.

At a little league game, a seven-year-old boy was called out on strikes. The boy's father was so infuriated with the umpire's decision that he shouted in a loud voice, "Kill the umpire." The boy, who was still holding his bat, swung the bat at the umpire. The umpire ducked and the bat flew out of the boy's hands and struck a spectator, who was seriously injured. In a tort action by the umpire against the boy which of the following statements is correct? (A) The umpire could recover only on an as- sault theory. (B) The umpire could recover either on an assault theory or a negligence theory. (C) The umpire could recover only on a negli- gence theory. (D) The umpire could not recover.

(A) The umpire could recover on an assault theory. Even though the boy did not hit the umpire, his action still constitutes an assault because it can be shown that he intended to cause harmful contact and actually created an apprehension of contact. Children are liable for their intentional torts when they are capable of forming the requisite intent. For assault, the intent required is to bring about the offensive or harmful contact; knowledge of its wrongfulness is not an element of that intent. The fact that he swung the bat at the umpire strongly indicates that he believed his bat would hit the umpire and that the boy's purpose in doing so was to bring about this result. There- fore, it can be assumed that the boy had the capacity to form an intent to hit and that he did form that intent, which is a required element of the tort of assault. Thus, (C) and (D) are incorrect. The umpire could not recover on a negligence theory because the facts do not suggest that the umpire suffered any physical harm as a result of the boy's act. Specifically, most jurisdictions hold that recovery for negligent infliction of mental distress can only be had when the plaintiff has actually sustained physical contact as a result of defendant's actions or has suffered actual physical harm. These facts do not indicate that the harm done to the umpire is the kind of harm compensable in negligence. Therefore, (B) and (C) are incorrect.

Which of the following is correct regarding self-defense? A Retaliation may be permissible. B Deadly force may be permissible. C Retreat is required. D Actual necessity is required.

(B) Deadly force may be permissible for self-defense. A person may use deadly force to prevent death or serious bodily injury to herself. Self-defense requires use of force that reasonably appears necessary to prevent harm. A person may use deadly force if she reasonably believes that she is in danger of serious bodily injury. Retaliation is not permissible for self-defense. Self-defense is only permitted to prevent the commission of a tort. A person cannot retaliate by using force when there is no longer a threat of injury. Retreat is not required for self-defense. A majority of courts hold that a person may stand her ground and need not attempt an escape. Actual necessity is not required for self-defense. A person need only have an apparent necessity to defend oneself, i.e., a reasonable belief that she is being, or is about to be, attacked.Thus, a reasonable mistake as to the need for self-defense does not eliminate this defense.

Which one of the following types of common law defamation requires pleading and proof of special damages? A Libel not falling into one of the four per se categories. B Slander not falling into one of the four per se categories. C Slander not defamatory on its face. D Libel not defamatory on its face.

(B) Slander not falling into one of the four per se categories requires pleading and proof of special damages (i.e., pecuniary losses). Injury to reputation is not presumed for spoken defamation unless it falls into one of the four categories of slander per se. Damages will be presumed if the defamation (i) disparages the plaintiff in the conduct of her business or occupation; (ii) asserts that the plaintiff is suffering from a loathsome and communicable disease; (iii) alleges that the plaintiff has committed a serious crime or crime of moral turpitude; or (iv) imputes unchastity to a female plaintiff. Libel does not require special damages, even libel not falling into one of the four per se categories. In most jurisdictions, general damages are presumed by law for all libel; i.e., special damages need not be established. The fact that slander is not defamatory on its face does not affect whether special damages are required; rather, whether the slander falls into the per se categories determines that requirement. Only a minority of courts require special damages for libel not defamatory on its face. As stated above, most states do not require special damages for libel.

A landowner owned several dozen acres of mountain land near a national forest. A plaintiff who was injured by a condition on the owner's land brought an action for personal injury against the landowner. In a jurisdiction that applies the traditional rules for landowners and possessors of land, which of the following plaintiffs is most likely to win? (A) A 10-year-old trespasser who was swept onto some rocks while attempting to cross a swiftly flowing river. (B) A five-year-old trespasser who fell into a mineshaft from which the owner had removed all warning signs, but the plain- tiff was not attracted onto the owner's land because of the mineshaft. (C) A five-year-old trespasser who inadver- tently stepped into a badger hole that was obscured in the undergrowth. (D) A 10-year-old niece visiting the landowner who stepped into a badger hole that the landowner did not know was present but that could have been discovered by inspec- tion.

(B) The child who fell into the mineshaft is most likely to win. Under the attractive nuisance doctrine, a child trespasser who is injured by a dangerous artificial condition need not have been attracted onto the property by the condition. (A) is wrong because generally there is no obligation for a landowner to warn trespassers, whether they are children or adults, of dangerous natural condi- tions. (C) is wrong for the same reason. (D) is wrong because, as a licensee, the plaintiff need only be warned of dangerous natural conditions of which the landowner is in fact aware and which are unknown to the licensee or unlikely to be discovered by her, and here the landowner did not know of the hole.

A famous comedian was asked by the host of a popular late night television talk show what brand of cigars he smoked. He responded, "I smoke only [the manufacturer's] cigars, because they're the best." Two weeks later the manufacturer of those cigars began a national advertising campaign featuring billboards, posters for use in retail stores, and full-page ads in high circulation magazines. The advertising featured a picture of the comedian with the manufacturer's cigar in his hand, and the copy quoted his statement from the show. The manufacturer had not received the comedian's permission to use either his picture or the statement that had been made during the interview. Will the comedian prevail in an action against the manufacturer for using his picture and state- ment? (A) Yes, because the comedian has been de- famed. (B) Yes, because the comedian's likeness was appropriated for a commercial purpose without his consent. (C) No, because the advertising accurately reflects what the comedian said publicly before millions of television viewers. (D) No, because the comedian's appearance on television created an implied consent to reasonable use of anything he might say.

(B) The comedian will prevail on invasion of privacy grounds. (B) is a correct statement of the law defining the invasion of the comedian's privacy by appropriating his likeness for a commercial purpose (i.e., using the comedian's picture to promote the sale of a commercial product without the comedian's consent). (A) is incorrect because the comedian lacks a prima facie case for defamation (there was no defamatory statement causing damage to the comedian's reputation) and, in any case, what the manufacturer printed was true. (C) is relevant to defamation, but is incorrect here, because accuracy is not a defense to privacy torts. (D) is incorrect because consent to commercial appropriation must be specific; it may not be implied from the comedian's state- ments on the television program.

A horse breeder owned a small but excep- tionally well-tended horse farm for many years. The county in which the farm was located had no zoning or land-use regulations, but that had never been a problem until a half-acre plot of land next to the farm was recently purchased by a salvage company. The company let the weeds grow high on the land and it became littered with smelly, unsightly garbage and rusting metal. The breeder complained to the company on several occasions but was ignored. In addition, business started to taper off at the breeding farm due to the noise, smells, and general disarray of the junkyard. If the breeder brings an action for nuisance against the company, how will the court rule? (A) For the breeder, because the breeder was a property owner in the area long before the company bought the lot and opened the business. (B) For the breeder, if he can show a substantial and unreasonable interference with the use and enjoyment of his land. (C) For the company, because it is using the land for legal purposes. (D) For the company, unless the breeder can objectively demonstrate that the value of the farm has declined.

(B) The court will rule for the breeder. A private nuisance is a substantial, unreasonable interference with another's use or enjoyment of his own property; hence, (B) is correct because it states an element that the breeder must prove. (A) is incorrect because the fact that the plaintiff owned the property first does not establish the nuisance case. (C) is also incorrect. The fact that the defen- dant is using his property for legal purposes is not a defense to a nuisance action. (D) is incorrect, because a decline in the property's value is not an element of the prima facie case of nuisance.

A driver on a 3,000-mile cross-country trip in his new car tried to drive the entire trip without stopping, but fell asleep at the wheel, causing the car to strike a bridge abutment and roll over. The driver was seriously injured by the rollover, and suffered additional injuries when the turn signal rod broke off and punctured his lung. The driver had purchased the car from his local auto dealer. The car was manufactured by a local manufacturer, and the turn signal rod was manufactured by a subcontractor whom the manufacturer had used for many years. Tests after the accident established that the turn signal rod was defective and that the defect was the reason it broke off. The defect was not discov- erable through reasonable inspection and the manufacturer had had no prior indication of any defects. The driver brought a strict liability action against the manufacturer in a jurisdiction that does not apply its comparative negligence rules to strict liability actions. What is the likely result of the driver's action? (A) The driver will be awarded damages for all injuries incurred as a result of the accident. (B) The driver will be awarded damages for injuries incurred because the turn signal rod was defective, but he will not recover for other injuries incurred in the accident. (C) The manufacturer will prevail, because the accident was caused by the driver's negli- gence. (D) The manufacturer will prevail, because the driver cannot show that the manufacturer knew or should have known that the turn signal rod was defective.

(B) The driver will be awarded damages for injuries from the turn signal rod. Even though the defec- tive turn signal rod was not the proximate cause of the accident, the driver can recover from the manufacturer for the additional injuries suffered due to the defective turn signal rod, because those are identifiable as caused by that defect. The manufacturer is strictly liable under a products liability theory for injuries caused by that defect, even though a subcontractor manufactured the rod. Thus, (B) is the correct answer. (D) can be eliminated because it deals with a negligence theory, and the suit is being filed under a strict liability theory. Because liability attached only for the damages actually caused by the defective turn signal, (A) is incorrect.

An infant was injured in an automobile accident when the vehicle, driven by the infant's mother, left the roadway and rolled over down an embankment. At the time of the accident, the infant was buckled into an infant carrier car seat. The carrier was designed to snap into a base that was secured in the back seat by the rear center seat belt. Prior to driving, the mother had snapped the car carrier onto the base and pulled up on the car carrier's handle to ensure that the carrier was indeed secured in the base. When the rollover occurred, however, the carrier came loose from the base and was thrown about the inside of the vehicle, causing injuries to the infant's neck and face. The mother brought a products liability action on behalf of the child against the manufacturer of the car carrier, alleging that the manufacturer was negligent in the design of the base and seat combination. If the mother establishes at trial that the force of the rollover was enough to knock the seat loose, and that a reasonable, economically feasible alternative design existed, which of the following, if true, would be most helpful to the manufacturer's defense? (A) The mother violated a statute by traveling too fast for conditions, which caused the rollover accident. (B) No one had reported a car carrier coming loose in a rollover prior to this accident. (C) The car seat conformed with federal labeling requirements. (D) The retailer who sold the car seat was negli- gent in failing to notice the defect.

(B) The most helpful fact is that no one had reported this type of problem previously. The mother is alleging that the manufacturer's negligence led to the supplying of a defective product. To estab- lish this, the plaintiff must show that those designing the product knew or should have known of enough facts to put a reasonable manufacturer on notice about the dangers of marketing the product as designed. Negligent design is not shown, however, if the danger of the product becomes apparent only after the product reaches the public. Hence, the absence of any previous complaints about this problem would be most helpful to the manufacturer. (C) is less helpful than (B). Although compliance with government safety standards, such as labeling, is evidence that the product is not defective, it is not conclusive evidence, and federal labeling requirements do not preempt state products liability laws on defective warnings. (A) will not support the manufac- turer's defense. The answer choice suggests that the mother was contributorily negligent; however, the contributory negligence of a parent is not imputed to the child. (D) does not help the manufac- turer. An intermediary's negligent failure to notice a defect does not relieve the liability of a manufacturer whose original negligence was the cause of the defect.

A tenant moving into a new apartment bought a spool of "10-pound test" fishing line, manufactured by a fishing tackle and accessories company, for the purpose of hanging pictures, all of which she knew weighed less than 10 pounds. The spool came with no guidelines or warnings about using it for hanging objects. She attached the fishing line to either end of the pictures and hung them on hooks on the wall. The next week, a friend visiting the tenant was hit and injured while sitting on the couch by a picture that fell when the fishing line broke. It is common knowledge in the sporting goods industry that "10-pound test" indicates that the line will stand a pull of 10 pounds, but is not intended to support a 10-pound weight over a period of time. However, it is also common knowledge in the industry that the public in general uses fishing line to support heavy hanging objects over a period of time. The friend brought a products liability action against the tackle company for damages caused by his injury. What is the most likely result? (A) The friend will win, because the line failed to support a weight of less than 10 pounds. (B) The friend will win, because the label did not warn the consumer against relying on the term "10-pound test" for purposes other than fishing. (C) The friend will lose, because the line was not being used for its intended purpose, fishing. (D) The friend will lose, because the line conformed to the accepted standard for "10-pound test line."

(B) The most likely result is that the friend will prevail on grounds of inadequate warnings. Where a product presents an unreasonable risk of injury to users and bystanders, the fact that there is no sufficient warning of the danger may be a dangerous defect that will invoke strict products liability. The facts establish that the fishing tackle company knew that purchasers of its fishing line commonly used that line to support heavy hanging objects, and that the line would not support the weight rating in that type of use. Thus, the fishing tackle company's failure to provide a warning may be a defect that will permit the friend to recover in strict liability. Given that the other choices are clearly incorrect, (B) is the best option. (A) is incorrect because the fact that the line would not support a hanging weight of less than 10 pounds is not alone sufficient to constitute a dangerous defect; it is the fact that the fishing tackle company knew of the danger represented and failed to provide a warning to such users that may make the injury to the friend actionable. (C) is incorrect because the foreseeable misuse of a dangerously defective product by a purchaser or user does not relieve the manufacturer of the product of liability for injuries that are caused by the defect. Here, the known use of fishing line to support heavy hanging weights is what prompts the need for a warning, the absence of which may make the product dangerously defective. (D) is incorrect because, as indicated above, the presence or absence of a dangerous defect, and the responsibility for injuries caused by the defect, is not controlled by whether a product meets standards of performance measured solely by the use intended by the manufacturer. The fact that the fishing line performed adequately under industry standards when used for fishing does not mean that it cannot be dangerously defective when used for another purpose, when such use is known or reasonably should have been known by the manufacturer.

The parents of an eight-year-old boy at a public school told the school principal that their son said he had been molested by his teacher. The next day, the principal confronted the teacher with the accusation. The teacher admitted sexually molesting the boy, but he pleaded with the principal for another chance, promising never to do it again. Although the principal believed him, he still insisted that the teacher must leave. The teacher then threatened to blackmail the principal with information he had about a former student of the principal who had an affair with him when he taught at the local high school. Fearful that he would lose his position if this accurate information ever came to light, the principal allowed the teacher to continue teaching, and told the family that the boy's charges were unfounded. Two weeks later, the teacher molested the boy again, and the teacher was arrested and charged with both incidents. In a suit against the principal on the boy's behalf, will the parents prevail? (A) Yes, because the principal is vicariously liable for the teacher's torts. (B) Yes, because the principal was aware that the teacher was a child molester and did nothing. (C) No, because the principal did not molest the boy. (D) No, because the principal believed the teacher when the teacher told the principal that he would not molest children again.

(B) The parents will win because the principal was aware of the teacher's conduct and did nothing. The principal and the teacher are in an employer/employee relationship. Normally, intentional torts of the employee are deemed not within the ambit of the employment relationship, except in jobs such as that of a bill collector or club bouncer. There is no evidence that intentional tortious conduct is part of this employment relationship. So the principal cannot be vicariously liable for the teacher's intentional torts and (A) is incorrect. However, there is a cause of action against the principal under a negligence theory. The principal knew that the teacher was a child molester. The principal initially indicated that the proper thing to do was for the teacher to leave. He backed down only under threats. Clearly action of some sort should have been taken, and the parents should have been told the truth about the teacher. The fact that the principal did nothing is clearly negligence on his part. (C) is incorrect because, as explained above, the principal can be liable for negligence even if he did not molest the boy. (D) is incorrect because, even though the principal believed the teacher, he had a duty to take action to protect the schoolchildren from the teacher.

Two members of a backgammon club owned identically sized, red backgammon boards. The first member's board was made of cheap material while the second member's board was quite expensive. One night, after a competitive tourna- ment, the two members met in the finals, playing on a borrowed board. The second member won and the first member, visibly upset, mistakenly grabbed the other's board and drove home. As was her custom, she left the board in the trunk of her car. Meanwhile, the owner of the board discovered the board switch and drove to the first member's apartment to make an exchange. The first member took the second to her parking place and saw that her car had been stolen. The police recovered the car days later, with no backgammon board in the trunk. The second member demanded a replacement board, but was refused. In an action to recover the board's value, will the second member recover? (A) Yes, because when the first member took the board she committed a trespass to chat- tel. (B) Yes, because when the board was stolen along with the car, the first member became liable for conversion. (C) No, because the first member believed in good faith that the board was hers when she took it from the club. (D) No, because the board was lost through no fault of the first member.

(B) The plaintiff will win because the defendant committed a conversion. A conversion occurs when the defendant intentionally causes serious interference with the chattel of the plaintiff. The intent involved refers to the physical act that results in the conversion, not to the defendant's desires regarding the ultimate disposition of the property. Therefore, the first member was guilty of conversion when she intentionally (i.e., volitionally) took the second's board, which resulted in its loss, even though the first member did not intend to lose it or even realize that she had taken the property of another. (A) is not the best answer because complete loss of a chattel is too serious an interference to be classified a mere trespass. (C) is wrong because the first member's good faith is irrelevant in a conversion action. (D) is wrong because the fact that the first member's car was stolen does not relieve her of liability.

A man and a woman who were fierce business competitors were both competing for a large job. The man submitted his bid and then went to the woman's office and told her, "If you leave this office, I'm going to get you!" The woman merely laughed and said, "I'm about finished with my bid and will be leaving in a few minutes." The man left the office but placed a large, heavy couch across the entrance to the woman's office, hoping to keep her from leaving. Meanwhile, the woman finished the bid and tried to leave her office, but found that she could not open the door. She pushed against the door as hard as she could and was eventually able to force it open, then ran all the way to the place where bids were being taken and got her bid in with one minute to spare. As usual, her bid was slightly lower than the man's, and she was awarded the contract. If the woman sues the man, what causes of action can she assert? (A) Assault, but not false imprisonment. (B) False imprisonment, but not assault. (C) Both assault and false imprisonment. (D) Neither assault nor false imprisonment.

(B) The woman has a cause of action for false imprisonment only. The woman has no cause of action for assault, because there was no act by the man that created a reasonable apprehension in the first person of immediate harmful or offensive contact. The man's words, unaccompanied by any act, constitute at most a threat of future contact. Such a threat is insufficient to create the requisite apprehension of immediate harmful or offensive contact. Because there is no assault, (A) and (C) are incorrect. The woman has a cause of action for false imprisonment because the man placed a physical barrier across the entrance to the woman's office, intending to confine the woman therein. The man's action caused the woman to be confined to the office. The fact that the woman apparently was confined for only a short time is immaterial to the false imprisonment action, as is the apparent absence of actual damages. Thus, (B) is correct and (A) and (D) are incorrect.

A tenant invited a friend over for dinner. On his arrival, the friend stepped on a split board on the front steps and the board broke, causing him to lose his balance and break his ankle. If the friend sues the tenant for his injuries and does not prevail in a jurisdiction that applies the traditional rules for landowners and possessors of land, what is the most likely expla- nation? (A) In the lease, the landlord had undertaken the duty to discover and repair dangerous conditions on the premises. (B) The friend arrived an hour earlier than his invitation specified. (C) The friend should have noticed the dangerous condition himself. (D) The tenant had stayed beyond the lease term and she no longer had the legal right to occupy the premises.

(C) If the friend does not prevail, it will be because he should have noticed the dangerous condition himself. In jurisdictions applying the traditional rules for landowners and possessors of land, the nature of the duty owed by an owner or occupier of land to those on the premises for dangerous conditions on the land depends on the legal status of the plaintiff in regard to the property, i.e., trespasser, licensee, or invitee. A licensee is one who enters on the land with permission for his own purpose or business and includes social guests. The owner or occupier owes a licensee a duty to warn of or make safe a dangerous condition known to the owner or occupier that creates an unreasonable risk of harm to the licensee and that the licensee is unlikely to discover. The owner or occupier does not have a duty to inspect for defects or to repair known defects. Here, the friend had been invited for dinner, making him a licensee. The facts do not indicate whether the tenant knew of the split board and neglected to alert the friend or simply was not aware of it, but the duty to warn does not extend to dangerous conditions that the licensee should reasonably have discov- ered. Hence, (C) presents the best basis for the friend not prevailing. (A) is incorrect because that fact would not make a difference to the tenant's liability. The tenant remains liable to the friend for dangerous conditions on the premises as the occupier of the land, regardless of the landlord's obligation to inspect and repair. (B) is incorrect because the friend still qualifies as a licensee even though he arrived sooner than his invitation specified. It is true that a person may lose invitee status and become a licensee by being on the premises at a time outside the scope of his invitation. However, there is no similar principle applicable here. The fact that the friend arrived an hour early does not make him a trespasser rather than a licensee. Hence, the fact in (B) would not affect the tenant's liability. (D) is incorrect because it also is irrelevant. Even if the tenant had no legal right to occupy the land, she still would be the possessor of the land as to the friend, and she owed the friend the duties owed to a licensee.

A teenager who was totally blind in one eye and had only 10% vision in the other could not obtain a driver's license. Nevertheless, on his 18th birthday, he borrowed his father's car and took his girlfriend for a ride. With his 10% vision in one eye, he was able to stay in the correct lane and avoid oncoming traffic, but he failed to see a jogger on the edge of the highway. The teenager's car hit the jogger, causing serious bodily injury. If the jogger brings a negligence suit against the teenager and the jury finds in the jogger's favor, what is the most likely reason? (A) The teenager failed to exercise ordinary and reasonable care under the circumstanc- es. (B) The teenager failed to exercise the amount of care that an 18-year-old of like educa- tion, intelligence, and experience would have exercised. (C) The teenager failed to exercise the ordinary and reasonable care that a person with the teenager's disability would have exercised. (D) Theteenagerviolatedthelawwhenhe drove without a license.

(C) If the jogger prevails, it will be based on failure to exercise ordinary care of someone with the teenager's vision problem. In a lawsuit based on negligence, the usual standard of care is ordinary and reasonable care under the circumstances. The standard changes when the defendant has a major physical disability such as blindness. In that situation, the standard becomes ordinary and reasonable care for a person with that disability. Therefore, (C) is a more accurate answer than (A). (B) is wrong because it states the standard to be applied to children, which would not be applied to an 18-year-old car driver. (D) is a true statement. However, the statutory violation of failure to obtain a license was not the cause of the accident.

A trucker owned and operated a small truck which he used commercially to haul dynamite to construction sites. Unbeknownst to the trucker, there was a hidden defect in the latch that held the rear panel of the truck. The trucker was hauling a load of dynamite one morning and exceeding the speed limit when his truck struck a bump in the road, the latch malfunctioned, and the rear panel of the truck flew open. One box of dynamite fell out of the truck and struck a pedes- trian, breaking her foot. If the pedestrian sues the trucker under strict liability for her injuries, will the pedestrian win? (A) Yes, because hauling dynamite is an abnor- mally dangerous activity. (B) Yes, because the trucker was speeding while driving with the dynamite. (C) No, because the defect in the latch was not discoverable upon reasonable inspection. (D) No, because the dynamite did not explode.

(D) The pedestrian will lose in this suit. The defendant is strictly liable for engaging in certain activities when the dangerous propensity of that activity injures the plaintiff. Although hauling dynamite is an unusually dangerous activity, it was not the dynamite's dangerous propensities that caused the accident. Instead, it was a defective latch in the truck. Thus, (A) is incorrect. (B) and (C) can be eliminated because they deal with negligence issues and the pedestrian is suing under a strict liability theory. (D) is the best answer. Note that if the dynamite had exploded, after falling out due to the defective latch, there would have been liability under a strict liability theory.

A driver traveling the speed limit in the evening on a quiet country road rounded a curve and struck a bicyclist who was riding in the same lane. The driver stopped the car and inspected the bicyclist, who had a broken leg. The driver thought it best not to try to move the bicyclist, so he told him that he would go to get help. The driver drove away and left the bicyclist by the side of the road. After the driver had left the scene, he realized that he had forgotten his wife's birthday, so he stopped to buy a gift and hurried home. He did not remember the bicyclist until a few hours later, but assumed that by that time someone would have come along to render assis- tance. However, the bicyclist was not rescued until the following morning. By then, he had contracted pneumonia as a result of exposure. The bicyclist sued the driver to recover damages for his broken leg and the pneumonia. If the jury finds that the driver was not negli- gent in his operation of his automobile, for what harm will the bicyclist most likely recover? (A) Both the leg injury and the pneumonia. (B) The leg injury but not the pneumonia. (C) The pneumonia but not the leg injury. (D) Neither the leg injury nor the pneumonia

(C) The bicyclist will most likely recover for the pneumonia but not for the leg injury. The facts indicate that the driver was not driving negligently when the accident occurred. Therefore, he is not liable for the leg injury caused by the accident, and (A) and (B) are incorrect. However, where the defendant's actions have placed another person in peril or caused another's injury, the defendant has a duty to make reasonable efforts to rescue the imperiled person or render aid to his victim. The driver's neglect of the bicyclist after injuring him will make him liable for the resulting pneumonia. Therefore, (B) and (D) are incorrect.

A camper at a state park built a campfire within a fire ring on a calm day according to approved procedures. Just as a sudden strong wind arrived and blew some embers onto the grass, a large bear came out of the woods and charged at the camper. The camper ran to his car, which was some distance away, with the bear in close pursuit. By the time the bear left and the camper was able to exit the car and summon assistance, the embers in the grass had started a brush fire. The fire destroyed another camper's equipment and automobile at a nearby campsite before it could be extinguished. The other camper sued the camper who started the fire. At trial, the parties stipulated to the above facts. The plaintiff introduced into evidence a state statute that prohibited leaving any campfires unattended and required them to be extinguished immediately if any embers were blown out of the fire ring. At the conclusion of the proofs, both parties moved for a directed verdict. How should the court rule on the motions? (A) Grant the plaintiff's motion, because the statute was intended to prevent the type of harm that occurred, making the statutory standard applicable. (B) Grant the plaintiff's motion, because a brush fire caused by a campfire does not ordinarily happen in the absence of negli- gence by the camper. (C) Grant the defendant's motion, because the plaintiff has not established a prima facie case of negligence. (D) Deny both motions, because the jury should make the factual determination of whether the defendant was negligent.

(C) The court should grant the defendant's motion because the plaintiff has not offered sufficient evidence of negligence on the defendant's part to go to the jury. The standard of care in a negli- gence case may be established by proving that a statute imposing a specific duty applies instead of the more general common law duty of care. However, violation of the statute may be excused where compliance would cause more danger than violation or where compliance would be beyond the defendant's control. Here, the statute regulating campfires is applicable because (i) the plain- tiff, a fellow camper, is in the class intended to be protected by the statute, (ii) the statute was designed to prevent the escape of a campfire, which is what occurred here, and (iii) the statute clearly specifies what is required. However, even though the statute would apply to the defendant's conduct and the defendant violated the statute, the violation will be excused here because he was fleeing for his life from a bear and had to take refuge in his car. Hence, the defendant will not be held to the statutory standard of care here. Because the plaintiff has offered no other evidence that the defendant was negligent, the defendant's motion should be granted. (A) is incorrect because, as discussed above, even though the statute was intended to prevent the harm that occurred, the defendant's violation of the statute will be excused. (B) is incorrect because the plaintiff cannot rely on an inference of negligence here to establish breach of duty. Res ipsa loquitur permits the trier of fact to infer breach of duty where the facts strongly indicate that the plaintiff's injuries resulted from the defendant's negligence, but here the undisputed facts as to how the brush fire was caused are inconsistent with a finding of negligence. (D) is incorrect because there is neither a reasonable inference of negligence nor evidence of negligence, given that the violation of the statute will be excused here. Hence, the case should not be submitted to the jury because the plaintiff has not established a prima facie case.

A defendant intended to commit an assault on A, but his conduct only constituted a battery on B. Under the transferred intent doctrine, the defendant is liable for: A An assault of B. B An attempted assault of A and a battery of B. C A battery of B. D An attempted assault of A and an assault of B.

(C) The defendant has committed a battery of B when he acts with the intent to commit an assault on A, but his conduct only constitutes a battery on B. The transferred intent doctrine allows an intent to commit a tort against one person to be transferred to the committed tort or to the injured person. It applies to (i) assault, (ii) battery, (iii) false imprisonment, (iv) trespass to land, and (v) trespass to chattels. The defendant is not liable for an assault of B. The committed tort was a battery, and the intent transfers from the assault to the battery. Nor is the defendant liable for an attempted assault of A. There is no tort liability for an attempted assault standing alone. The defendant is liable only because of the transferred intent doctrine, and only to the person harmed. Thus, there is no liability to A here.

A hunter constructed an outfit which made him appear, from the waist up, to be a 16-point buck. Below the waist he wore camouflage pants and hiking boots. The day hunting season opened, the hunter stationed himself in a heavily populated deer area and awaited his prey. Meanwhile, a second hunter, who had had numerous alcoholic drinks before embarking on opening day hunting, caught sight of the first hunter in his deer costume and fired his shotgun. One of the bullets grazed the first hunter's head, blasting the deer mask off and knocking the hunter unconscious. The first hunter brings an action to recover damages for his personal injuries against the second hunter in a jurisdiction retaining tradi- tional contributory negligence rules. What result? (A) The second hunter will win, because he did not have the requisite intent to justify imposing liability for an intentional tort on the first hunter. (B) The second hunter will win, if he can prove that he did not have the last clear chance to avoid injuring the first hunter. (C) The first hunter will win, if he can estab- lish that the second hunter's act of hunting while intoxicated was reckless and wanton conduct. (D) The first hunter will win, if he can show that the second hunter was negligent in hunting while intoxicated.

(C) The first hunter will win if the second hunter's conduct was reckless or wanton. Because the first hunter was contributorily negligent in making himself appear to be a deer on opening day of deer hunting season, his recovery depends on the effect of his contributory negligence. Without comparative negligence principles, the first hunter's contributory negligence might bar his recovery unless he can show that the second hunter was reckless and wanton, as to which conduct contributory negligence is no defense. Thus, (C) is the correct pick. (A) is wrong because the second hunter's liability is not dependent on intentional tort theories. (B) is not the best answer because, even if the second hunter did not have the last clear chance (which overcomes contribu- tory negligence), he may still be liable if his actions are characterized as reckless and wanton. (D) is wrong because, absent the factors discussed above, the second hunter's negligence would be counterbalanced by the first hunter's contributory negligence.

What is the key difference between trespass to chattels and conversion? A The value of the chattels that are subject to the defendant's interference. B The nature of the intent on the part of the defendant. C The seriousness of the interference with the plaintiff's possession. D The means by which the defendant interferes with the plaintiff's possession.

(C) The key difference between trespass to chattels and conversion is the seriousness of the interference with the plaintiff's possession. An act by the defendant interfering with the plaintiff's right of possession in the chattel that is serious enough in nature or consequence to warrant that the defendant pay the full value of the chattel constitutes conversion. On the other hand, a less serious interference with the plaintiff's right of possession will constitute trespass to chattels (as long as the plaintiff suffered some damage). No specific rule can be stated for these situations; however, the longer the withholding period and the more extensive the use of the chattel during this time, the more likely it is that conversion has resulted. The value of the chattels that are subject to the defendant's interference is not determinative. The defendant may interfere with the plaintiff's possession of a valuable chattel but only briefly or in a minor way, and the defendant is liable only for trespass to chattels. The nature of the intent on the part of the defendant is essentially the same for both torts: the intent to perform the act that interferes with the plaintiff's right of possession. The defendant need not have intended to cause a serious interference to be liable for conversion. The means by which the defendant interferes with the plaintiff's possession need not be different for the two torts; it may be the consequences of the interference that make the difference. For example, a short joyride may amount to only trespass to chattels, but if the defendant accidentally totals the car while doing so, he will be liable for conversion.

After accounts of a confidential congres- sional hearing on a national security matter were published, the chief counsel at the hearing made a statement to a major newspaper accusing a popular network news anchorman of leaking the story and endangering national security. The network immediately fired the anchorman. When facts came to light a few weeks later showing that the allegation was not true, the anchorman was rehired and restored to his position. The anchorman sued the newspaper for defamation, claiming compensatory and punitive damages, and made allegations legally sufficient to sustain those damages if proved. No affirma- tive defenses were allowed. What is the newspaper's best defense? (A) It was not negligent in printing the chief counsel's remarks. (B) The anchorman was restored to his position within a few weeks. (C) The publication was not made with knowl- edge that it was false or with reckless disre- gard for the truth. (D) The statement was protected by the Speech and Debate Clause.

(C) The newspaper's best defense is that the publication was not made with actual malice. A public figure suing for damages in a defamation action must prove actual malice. Mere hostility or dislike of the plaintiff does not constitute actual malice. To establish actual malice, a plaintiff must show "that the utterance was false and that it was made with knowledge of its falsity or with reckless disregard of whether it was false or true." [New York Times v. Sullivan (1964)] Reckless disregard is not measured by whether a reasonable person would have investigated before publishing. There must be sufficient evidence to permit the conclusion that the defendant in fact entertained serious doubts as to the truth of the communication when it was published. (A) is incorrect because negligence is not the standard which will apply in this case to determine the defendant's fault. As the anchor of a popular news show, the anchorman is a public figure, and a public figure suing for damages in a defamation action must prove actual malice. (B) is incorrect because the fact that the anchorman was restored to his position quickly is relevant to the issue of damages, but it does not help the newspaper on the issue of liability. (D) is incorrect. The Speech and Debate Clause provides immunity from defamation, but only for communications in one of the houses of Congress. The chief counsel's statement outside of Congress would not be covered.

A pedestrian crossed the street at a crosswalk without looking for oncoming traffic. He was struck first by a car and then by a truck. The pedestrian sued both the driver of the car and the driver of the truck for negligence. The jury determined that the pedestrian was 60% at fault, the driver of the car 30%, and the truck driver 10%. The jury also determined that the pedes- trian suffered damages of $100,000. The driver of the car is insolvent. In a pure comparative negligence jurisdiction retaining traditional joint liability rules, how much can the pedestrian collect from the driver of the truck? (A) Nothing. (B) $10,000. (C) $40,000. (D) $100,000.

(C) The pedestrian can collect $40,000 from the driver. In a pure comparative negligence jurisdic- tion, the plaintiff can recover even if he was over 50% at fault. Thus, (A) is wrong. The recovery will be limited to the percentage of damage attributed to the defendant(s), in this case 40%. The jurisdiction retains the rule on joint and several liability. Therefore, each defendant is responsible for the combined liability of all defendants. (C) is therefore correct and (B) is wrong. (D) is wrong because, in a comparative negligence jurisdiction, the plaintiff's recovery is reduced due to his negligence.

While returning from transporting a group of children to summer camp, a bus driver and his assistant were caught in the leading edge of a forest fire raging down the high mountains. Hurrying ahead of the flames and smoke, the driver reached the last half-mile of a dirt road that ran to the main highway and safety, but he discovered that the road ahead was already blocked by fallen, burning foliage. Separating the driver's bus from the main highway, which angled off to the right, was the fenced property of a rancher. The bus driver drove across the property to reach the main highway, damaging some turf and a fence, and proceeded to the city. If the rancher asserts a claim against the bus driver to recover for the damage to his property, is the rancher likely to win? (A) No, because the bus driver was acting to protect the lives of himself and his assis- tant. (B) No, because the bus driver acted as would any reasonably prudent person under the circumstances. (C) Yes, because the bus driver damaged the rancher's property when he drove through the fence to get to the main highway. (D) Yes, because the bus driver intentionally drove across the property, knowing it would cause damage.

(C) The rancher will probably win. A person who intentionally intrudes upon land in the possession of another is guilty of the tort of trespass to land. The bus driver has clearly met all the elements of this tort. However, the bus driver has the defense of private necessity, because it was necessary to drive onto the rancher's land to avoid the forest fire. Although private necessity is a defense to trespass to lands, it does not relieve the bus driver of liability for damage done to the property. Hence, (A) and (B) are incorrect. Note that the call of the question merely asks what the outcome of the rancher's claim against the driver would be, not whether the driver has committed a trespass. (D) is not as good a choice as (C) because it focuses on the driver's intent in committing a trespass rather than on whether the land was damaged.

o assert the defense of property, a defendant using force against another may not: A Use force that may injure the other. B Make a mistake about the right to use force. C Use force without a request to desist. D Use force against one with a privilege to enter the property.

(D) A defendant cannot assert the defense of property if she uses force against one with a privilege to enter the property. Whenever an actor has a privilege to enter upon the land of another because of necessity, right of reentry, right to enter upon another's land to recapture chattels, etc., that privilege supersedes the privilege of the land possessor to defend her property. It is not true that the defendant may not use force that may injure the entrant. The force used must be reasonable and not likely to cause death or serious bodily injury. It is also incorrect to state that the defendant may not use force without a request to desist. A request to desist must usually precede the use of force, but if the circumstances make it clear that the request would be futile or dangerous, then a request to desist is not required. It is also incorrect to state that the defendant may not make a mistake about the right to use force. A reasonable mistake is allowed as to the property owner's right to use force in defense of property, where the mistake involves whether an intrusion has occurred or whether a request to desist is required.

A physician performed scheduled surgery on her patient's right ear for a condition caused by prolonged and repeated infections in that ear. During the surgery, the physician determined that her patient had been particularly susceptible to this condition due to a previously unsuspected anatomical abnormality. The physician reason- ably believed that this same abnormality was likely to exist in the patient's left ear. Though the patient had not had many infections in the left ear, if a similar course of recurring infections were to transpire involving that ear, it would probably develop the same condition as the right and require surgery. The physician therefore decided to perform surgery on her patient's left ear, although she had received his consent only to operate on the right ear. The surgery was performed with due care and was successful. In an action by the patient against the physi- cian, what is the likely result? (A) The patient will not recover because the extension of the operation was successful. (B) The patient will not recover because the extension of the operation was carried out with due care. (C) The patient will recover at least nominal damages on a negligence theory. (D) The patient will recover at least nominal damages on a battery theory.

(D) Thepatientwillrecoveratleastnominaldamages.Thepatienthereneedonlyshowthattheexten- sion of the operation was an intentional, unpermitted, offensive contact in order to recover at least nominal damages in battery. The patient may recover in battery regardless of whether she was harmed. Battery is a tort where no physical harm need be shown, and no actual damage need be proven. (A) is incorrect because the patient here can show that the extension of the operation was an intentional, unpermitted, offensive contact. Therefore, the patient may recover in battery regard- less of whether or not she was harmed. Battery is a tort where no physical harm need be shown, and no actual damage need be proven. Actual harm is an element of the prima facie case of negli- gence, but is not required for most intentional torts. (B) is incorrect because the patient may recover in battery regardless of whether she was harmed and whether the physician's conduct was incompe- tent (i.e., negligent). Battery is a tort where no physical harm need be shown, and no actual damage need be proven. Also, no negligence or incompetence need be shown in battery. The patient here need only show that the extension of the operation was an intentional, unpermitted, offensive contact. (C) is incorrect because, in a negligence action, actual harm must be shown. The plaintiff must establish that she suffered damage as a result of the defendant's conduct. Damages will not be presumed and nominal damages cannot be awarded in a negligence action. Nominal damages are only appropriate in a suit based on an intentional tort where no actual harm can be shown.

The plaintiff was driving inattentively when she had to swerve to avoid two other negligently driven vehicles at a busy intersection, and her car struck a light pole. The plaintiff, who was the only driver injured, sued one of the other drivers to recover damages in a jurisdiction that has adopted pure comparative negligence. The jury determined that she suffered injuries of $100,000 and was 50% at fault. If the plaintiff is awarded a recovery of only $25,000 from the defendant, what will be the most likely reason? (A) The defendant's fault was less serious than that of the other tortfeasor. (B) The plaintiff's fault was as great as the total negligence of the other two drivers combined. (C) The jurisdiction applies contribution based on a pro rata approach rather than propor- tional fault. (D) The jurisdiction has abolished joint and several liability.

(D) If the plaintiff recovers only $25,000, it will be because the jurisdiction has abolished joint and several liability. Under joint and several liability, when two or more tortious acts combine to proximately cause an indivisible injury to a plaintiff, each tortfeasor is liable to the plaintiff for the entire damage incurred. Hence, the plaintiff could recover $50,000 from the defendant if joint and several liability applied (her total damages reduced by the amount of her own fault that contrib- uted to her injury). The facts do not state what percent of fault was assigned to the defendant, but given that the other choices are incorrect, the defendant's fault must have been determined to be 25%, so that $25,000 would be the plaintiff's recovery in the absence of joint and several liability. (A) is incorrect regardless of whether joint and several liability applies. If the defendant's fault were less serious than that of the other tortfeasor, he would be liable to the plaintiff for less than $25,000 in the absence of joint and several liability. If joint and several liability did apply, he would be liable to the plaintiff for $50,000—the full amount of her damages. (B) is incorrect because the fact that the plaintiff's fault equals the combined fault of the other two tortfeasors is relevant only in a partial comparative negligence jurisdiction. It is irrelevant in a pure comparative negligence jurisdiction. (C) is incorrect because the rule of contribution, regardless of whether it is based on a pro rata approach or a proportional fault approach, does not affect how much the plain- tiff receives from a defendant. Rather, it enables a defendant who has paid more than his share of damages to the plaintiff under joint and several liability to seek recovery against any other joint tortfeasor for the excess paid.

A college student was holding a loud party at her house. The next door neighbor was getting increasingly angry with the noise, and several complaints to the local police station brought no results. The neighbor called the student on the phone and told her that if she did not stop the noise, he would "come over there and cut your throat." The visibly shaken student told her guests what had just happened, and they all decided to leave immediately. The student was unable to sleep that night and thereafter purchased an alarm system for her house and a gun that she kept next to her bed. If the student brings an action for intentional infliction of emotional distress and succeeds, what is the most likely reason? (A) The neighbor had the apparent present abil- ity to make good on his threat. (B) The student suffered some physical harm as a result of her distress. (C) The student suffered pecuniary injury as a result of the neighbor's threat. (D) The neighbor's conduct was extreme and outrageous.

(D) If the student wins, it will be because the neighbor's conduct was extreme and outrageous. Inten- tional infliction of emotional distress requires proof that: (i) the conduct was extreme and outra- geous; (ii) it was also intentional, or at least reckless; and (iii) severe emotional distress resulted. If the threat is deemed to be extreme and outrageous, the student will prevail because the other elements are present. (A) is incorrect because an apparent present ability to make good on a threat is required for an action of assault, but it is not specifically required for an action for inten- tional infliction of emotional distress, the tort at issue here. (B) is incorrect because the require- ment of physical harm only applies in cases of negligent infliction of emotional distress. This is a case of intentional infliction of emotional distress. (C) is incorrect because pecuniary injury is not required in an action for intentional infliction of emotional distress. While severe emotional distress must be shown, an action will lie even if no actual monetary harm was caused.

Which of the following is NOT part of the prima facie case for intentional infliction of emotional distress? A Intent to cause the plaintiff to suffer severe emotional distress. B Evidence of severe emotional distress. C An act by the defendant amounting to extreme and outrageous conduct. D Physical symptoms caused by the emotional distress.

(D) Physical symptoms caused by the emotional distress are not required. A prima facie case for intentional infliction of emotional distress requires proof of: (i) an act by the defendant amounting to extreme and outrageous conduct; (ii) intent on the part of the defendant to cause the plaintiff to suffer severe emotional distress, or recklessness as to the effect of the defendant's conduct; (iii) causation; and (iv) damages—severe emotional distress. Intent to cause the plaintiff to suffer severe emotional distress establishes the intent element of the tort. Reckless disregard of a high probability that emotional distress will result also satisfies the intent element of the tort. Evidence of severe emotional distress must be shown; hurt feelings are not sufficient. An act by the defendant amounting to extreme and outrageous conduct is an element of the tort.

Two men stopped to have a drink together at a bar, where five other people were also gathered. The two men had a few drinks and began to argue, hurling insults at each other. After the first man threw a punch, a fight ensued, and the second man was struck in the back of the head with a heavy object from behind. He never saw who had struck the blow and he was knocked unconscious. He was hospitalized for a severe concussion and still suffers from severe headaches. The injured man sued the first man as well as the five other persons present in the bar at the time he was struck, seeking to recover damages for the blow to his head. He presented the evidence above. Will the injured man prevail? (A) Yes, because he was not the aggressor. (B) Yes, because the first man acted unreason- ably. (C) Yes, because of the doctrine of res ipsa loquitur. (D) No, because he cannot prove that there was concerted action against him.

(D) The injured man will not prevail. While concerted action would create joint and several liability among all parties sued by the injured man, he has presented no evidence of this and therefore cannot prevail. (A) and (B) are incorrect. The fact that the first man was the aggressor or acted unreasonably does not, by itself, impose liability on the other persons sued. (C) is incorrect because res ipsa loquitur does not apply when more than one party may have been in control of the instrumentality causing injury. It requires that the instrumentality causing the injury be in the defendant's sole control. Furthermore, res ipsa loquitur creates a prima facie case for negligence, and the striking of the second man involves an intentional tort.

A bystander who witnesses the defendant negligently injuring another can recover for negligent infliction of emotional distress in most states by showing: A A close relationship between the bystander and the person injured, the bystander's presence at the scene of the injury, and the bystander's observation or perception of the event. B A close relationship between the bystander and the person injured, the bystander's presence within the zone of danger from physical injury, and the bystander's observation or perception of the event. C The bystander's presence within the zone of danger from physical injury, and the bystander's observation or perception of the event. D A close relationship between the bystander and the person injured, and the bystander's observation or perception of the event.

A bystander must show a close relationship between the bystander and the person injured, the bystander's presence at the scene of the injury, and the bystander's observation or perception of the event to recover. Traditionally, a bystander outside the "zone of danger" of physical injury who sees the defendant negligently injuring another could not recover damages for her own distress. A majority of states now allow recovery in these cases as long as (i) the plaintiff and the person injured by the defendant are closely related; (ii) the plaintiff was present at the scene of the injury; and (iii) the plaintiff personally observed or perceived the event. The bystander's presence within the zone of danger from physical injury is no longer required for a bystander to recover for witnessing an injury to another.

Which of the following best states who may bring a strict liability action against a defendant engaging in abnormally dangerous activities? A Anyone injured as a result of the dangerous propensity of the activity B Any foreseeable plaintiff injured as a result of the dangerous propensity of the activity C Anyone directly injured by the activity D Any foreseeable plaintiff as long as she was directly injured by the activity

A defendant engaging in an abnormally dangerous activity may be liable only to foreseeable plaintiffs injured as a result of the dangerous propensity of the activity. This is the best statement of the scope of the duty owed. In most states, a defendant will be liable only to those persons to whom a reasonable person would have foreseen a risk of harm under the circumstances. In general, strict liability is not imposed for injuries to a plaintiff to whom no reasonable person would have foreseen a danger. The defendant will not be strictly liable to all plaintiffs who were directly injured by the activity. Rather, the harm must result from the kind of danger to be anticipated from the abnormally dangerous activity; i.e., it must flow from the "normally dangerous propensity" of the activity involved. Conversely, a foreseeable plaintiff may recover even if she was not directly injured by the activity as long as the injury was from the dangerous propensity (e.g., injuries caused by fleeing the danger from the activity).

A plaintiff who is 40% negligent can recover 60% of his damages in which of the following cases? A In either a pure or a partial comparative negligence jurisdiction, regardless of the number of defendants and their degree of fault. B In a pure comparative negligence jurisdiction only. C In a partial comparative negligence jurisdiction, but only against a defendant who is more at fault than the plaintiff. D In a partial comparative negligence jurisdiction, but only if there is only one defendant.

A plaintiff who is 40% at fault can recover in either a pure or a partial comparative negligence jurisdiction, regardless of the number of defendants and their degree of fault. In a pure comparative negligence state, the plaintiff can recover no matter how great her degree of fault. In a partial comparative negligence jurisdiction, the plaintiff's recovery will be barred if her negligence passes a threshold level, either 50% or 51%. Hence, the plaintiff here can recover in a pure comparative negligence jurisdiction and in any partial comparative negligence jurisdiction. For that reason, it is incorrect to state that the plaintiff can recover in a pure comparative negligence jurisdiction only. In a partial comparative negligence jurisdiction, if several defendants have contributed to a plaintiff's injury, a "combined comparison" approach is used to determine the threshold level; i.e., the plaintiff's negligence is compared with the total negligence of all of the defendants combined. Thus, a plaintiff who is 40% at fault can recover in a partial comparative negligence jurisdiction even if there is more than one defendant, and may recover against a defendant who is also less at fault than she; the plaintiff will still be less at fault than the combined fault of the defendants.

A principal will be vicariously liable for the tortious acts of her independent contractor: A If the independent contractor is engaged in inherently dangerous activities. B Under the doctrine of respondeat superior. C If the principal negligently selected the independent contractor. D If the principal negligently supervised the independent contractor.

A principal will be vicariously liable for the tortious acts of her independent contractor if the independent contractor is engaged in inherently dangerous activities. In general, a principal will not be vicariously liable for tortious acts of an independent contractor. Two broad exceptions exist, however: (i) the independent contractor is engaged in inherently dangerous activities, e.g., excavating next to a public sidewalk, blasting; or (ii) the duty, because of public policy considerations, is simply nondelegable, e.g., the duty of a business to keep its premises safe for customers. Respondeat superior is the doctrine that makes employers vicariously liable for the torts of employees; it does not apply to independent contractors. A principal may be liable for negligently selecting or supervising an independent contractor. However, that liability is for her own negligence; it is not vicarious liability.

A duty of care is generally NOT owed to: A A rescuer, unless the defendant negligently put herself or a third person in peril. B A viable fetus. C A third party for whose economic benefit a legal or business transaction is made. D A discovered trespasser.

A rescuer is a foreseeable plaintiff and is owed a duty of care as long as the rescue is not wanton, but only if the defendant negligently put herself or a third person in peril. However, firefighters and police officers may be barred by the "firefighter's rule" from recovering for injuries caused by the risks of a rescue. A duty of care is owed to a viable fetus; prenatal injuries are actionable. A third party for whose economic benefit a legal or business transaction is made is owed a duty of care if the defendant could reasonably foresee harm to that party if the transaction is done negligently. While a landowner owes no duty to an undiscovered trespasser, a trespasser whose presence on the property is known to the landowner is owed a duty to be warned about seriously dangerous artificial conditions on the property.

In a negligence action, the plaintiff cannot recover: A Unforeseeable damages B Noneconomic damages C Presumed damages D Damages for lost future earning capacity

In a negligence action, the plaintiff cannot recover presumed damages. Damage is an essential element of a plaintiff's prima facie case for negligence. This means actual harm or injury. Unlike for some intentional torts, damage will not be presumed in negligence. A plaintiff is entitled to all damages that he can prove, even if the extent of the damages was unforeseeable. Permissible damages includes economic damages, such as medical expenses and lost earnings, and noneconomic damages, such as pain and suffering. The plaintiff is also entitled to damages for lost future earning capacity, discounted to present value to avoid an excess award; i.e., the plaintiff receives an amount that, if securely invested, would produce the income that the jury wishes him to have.

Which of the following describes only dependent intervening forces in a proximate cause analysis? A Efforts to protect person or property and acts of God. B Subsequent medical malpractice and criminal acts of third persons. C A subsequent disease and negligence of rescuers. D A subsequent accident and an intentional tort of a third person.

Dependent intervening forces are normal responses or reactions to the situation created by the defendant's negligent act. Dependent intervening forces are almost always foreseeable. A subsequent disease is a common dependent intervening force. The original tortfeasor is usually liable for diseases caused in part by the weakened condition in which the defendant has placed the plaintiff by negligently injuring her. Also, negligence of rescuers is a common dependent intervening force. Generally rescuers are viewed as foreseeable intervening forces, so the original tortfeasor usually is liable for their negligence. Efforts to protect person or property are common dependent intervening forces. A defendant is usually liable for negligent efforts on the part of persons to protect the life or property of themselves or third persons endangered by the defendant's negligence. Subsequent medical malpractice is also a common dependent intervening force. The defendant is usually liable for the aggravation of the plaintiff's condition caused by the malpractice of the treating physician. A subsequent accident may also be a dependent intervening force if the original injury was a substantial factor in causing the second accident. However, acts of God and intentional torts and criminal acts of third persons are independent intervening forces. Independent intervening forces operate on the situation created by the defendant's negligence, but they are independent actions rather than natural responses or reactions to the situation. (Note that the defendant may or may not be liable for independent intervening forces. It depends on whether they are foreseeable.)

In a products liability case based in strict liablity, a plaintiff may recover: A Only damages for economic losses B Only personal injury damages C Personal injury damages and property damages D Personal injury damages, property damages, and damages for economic losses

In a products liability case based on strict liability, a plaintiff may recover both personal injury damages and property damages for the supplying of a defective product. If the plaintiff's complaint is only that the product does not work as well as expected or requires repairs (i.e., no personal injury or property damages), most courts do not permit recovery of damages for economic losses under either a strict liability or a negligence theory; the plaintiff must bring an action for breach of warranty.

What must be shown for a qualified privilege to apply for defamatory statements? A There must be a common interest between the publisher and the recipient B The statement must be reasonably relevant to the interest being protected C The statement must not have been made with ill will D The statement must be in response to a request by the recipient

For a qualified privilege to apply, the statement must be reasonably relevant to the interest being protected. The privilege does not encompass the publication of irrelevant defamatory matter unconnected with the public or private interest entitled to protection. However, there need not be a common interest between the publisher and the recipient; statements made in the interest of one party but not the other may fall within a qualified privilege. While a qualified privilege does not apply if the defendant acted with actual malice, that term refers to knowledge of falsity or reckless disregard of truth rather than ill will; hence, a defendant acting with ill will may still assert a qualified privilege. Finally, the statement need not be in response to a request by the recipient; if the publisher has a relationship with the recipient, volunteered statements may fall within the privilege.

For assumption of risk to be available as a defense, the plaintiff must have: A Ignored the risk and negligently assumed it. B Known of the risk and voluntarily assumed it. C Known of the risk and expressly assumed it. D Ignored the risk and impliedly assumed it.

For assumption of risk to be available as a defense, the plaintiff must have known of the risk and voluntarily assumed it. A plaintiff who ignored a risk or negligently assumed it would be subject to the defense of contributory negligence rather than assumption of risk. For assumption of risk, it is irrelevant that the plaintiff's choice is unreasonable. While the plaintiff must have known of the risk, it need not be expressly assumed; it may be impliedly assumed. Different rules may apply, but both types of assumption of risk may be raised as a defense.

For a design defect in a products liability action, the plaintiff usually must show that: A A less dangerous modification or alternative for the product was economically feasible. B The product emerged from production different from other products and more dangerous than if it had been made the way it should have been. C The product was dangerous because it departed from its intended design. D The product does not comply with government safety standards.

For design defect cases, the plaintiff usually must show a reasonable alternative design, i.e., that a less dangerous modification or alternative for the product was economically feasible. Proof that the product emerged from production different from other products and more dangerous than if it had been made the way it should have been, or that the product was dangerous because of a departure from its intended design, establishes a manufacturing defect rather than a design defect. It is not necessary to show that the product does not comply with government safety standards to establish a design defect. A product may comply with the standards and still be found defective.

While practicing their target shooting at the firing range, a man and woman got into an argument that almost erupted into physical combat, except that they were restrained and separated by bystanders. Later, in the parking lot of the range, the man shot the woman in the shoulder. Bystanders who rushed to the scene immediately after hearing the man's shot found the woman on the pavement with a black metal flashlight in her hand. The woman's pistol was in her locker at the firing range. At the trial of the woman's civil action for battery against the man, the woman established that the man intentionally shot her. In defense, the man testified that the woman approached him, saying, "We'll settle this once and for all, right now," and raised an object toward the man. He testified that he feared that the woman was about to shoot him with a pistol, so he fired in self-defense. Assuming that the jury decides that the man is telling the truth, what must the jury also need to find for him to prevail? A No additional facts. B That a reasonable person in the same circumstances would have believed that the woman was about to shoot. C That the woman was at fault in raising a black object toward the man while threatening him. D That the woman was the original aggressor.

If the man prevails, it will be because the jury determined that he acted reasonably under the circumstances. One may act in self-defense not only where there is real danger but also where there is a reasonable appearance of danger. An honest but mistaken belief that the woman was about to shoot would justify the use of deadly force by the man if a reasonable person would have acted similarly under those circumstances. The test is an objective one—an honest belief alone is not sufficient. Thus, (A) is incorrect. (C) is incorrect because the woman's fault is not the determining factor—the reasonableness of the man's belief governs for self-defense. (D) is incorrect because it does not resolve whether the man had the right to use deadly force. Even if the man started the altercation at the range, he would have the right to use deadly force if the woman escalated the fight with deadly force.

There often is more than one cause for an injury. The "but for" test for actual cause applies to: A Joint causes. B Alternative causes. C Superseding causes. D Concurrent causes.

The "but for" test for actual cause applies to concurrent causes. An act or omission to act is the cause in fact of an injury when the injury would not have occurred but for the act. This test applies in concurrent cause cases, where several acts combine to cause the injury, but none of the acts standing alone would have been sufficient. But for any of the acts, the injury would not have occurred. The "substantial factor" test is used for joint causes, where several causes commingle and bring about an injury, but any one alone would have been sufficient to cause the injury. In that case, it is sufficient if defendant's conduct was a substantial factor in causing the injury. An alternative causes situation arises when two or more persons have been negligent, but uncertainty exists as to which one caused the plaintiff's injury. Under this approach, the plaintiff must prove that harm has been caused to him by one of them (with uncertainty as to which one). The burden of proof then shifts to the defendants, and each must show that his negligence is not the actual cause. Superseding causes arise in the context of proximate cause rather than actual cause. In addition to being an actual cause, the defendant's conduct must also be a proximate cause of the injury. Causes that arise after the defendant's conduct that contribute to the injury may be so unforeseeable as to be superseding causes, which cut off the defendant's liability for his original negligent act.

A bicyclist was riding his bicycle in the street when a negligently driven car struck the bike, knocking the bicyclist off the bike and breaking his right ankle. The driver of the car immediately stopped and went to his assistance. She got him to his feet and was slowly moving him toward the curb when a negligently driven taxicab struck him in the left leg. The bicyclist required surgery on both his right ankle and his left leg. If the bicyclist sues the driver and the cabbie, which of the following best states his right to recover? A He can recover from either the driver or the cabbie for all of his injuries because the driver and the cabbie are jointly and severally liable. B He can recover from the driver only for the injury to his right ankle and recover from the cabbie only for the injury to his left leg. C He can recover from either the driver or the cabbie for the injury to his left leg and recover from the driver only for the injury to his right ankle. D He cannot recover against the driver for the injury to his left leg unless the jury determines that the driver acted negligently when she came to his aid.

The bicyclist can recover from either party for the left leg injury but only from the driver for the right ankle injury. When two or more tortious acts combine to proximately cause an indivisible injury to a plaintiff, each tortfeasor is jointly and severally liable to the plaintiff for the entire damage incurred. Joint and several liability applies even though each tortfeasor acted entirely independently. However, if the actions are independent, plaintiff's injury is divisible, and it is possible to identify the portion of injuries caused by each defendant, then each will be liable only for the identifiable portion. Here, the cabbie would not be liable for the injury to the right ankle, because the cabbie did not cause the injury. (A) is therefore incorrect. With regard to the left leg, the cabbie was not the only cause of that injury. The original tortfeasor is liable for harm caused by the negligence of third persons when such negligence was a foreseeable risk created by the original tortfeasor's conduct. Here, as a result of the driver's original negligence, the bicyclist was in a position of danger while he was still in the street. The negligence of the cabbie in striking the bicyclist was a foreseeable risk while the bicyclist was in the street; it is therefore a foreseeable intervening force that will not cut off the driver's liability. Hence, both the driver and the cabbie will be jointly and severally liable for that injury. (B) is therefore incorrect. (D) is incorrect because the driver remains responsible for the foreseeable consequences of her original negligence in striking the bicyclist, regardless of whether she acted with due care when she came to his aid.

If the plaintiff establishes res ipsa loquitur, it will have the following effect: A A directed verdict will not be given for the defendant. B A directed verdict will be given for the plaintiff. C The burden of proof is shifted to the defendant. D A presumption of negligence is created.

The circumstantial evidence doctrine of res ipsa loquitur deals with those situations where the fact that a particular injury occurred may itself establish or tend to establish a breach of duty owed. Where res ipsa loquitur has been proven, the plaintiff has made a prima facie case, and a directed verdict will not be given for the defendant. Application of the doctrine does not shift the burden of proof to the defendant, nor does it create a presumption of negligence. Furthermore, the doctrine does not result in a directed verdict for the plaintiff. The defendant may introduce evidence that due care was exercised, and the jury may reject the permissible inference that may be drawn from the res ipsa proof and find for the defendant.

A small cruise ship struck a whale swimming underwater, causing the ship to suddenly lurch sideways. A passenger on the ship who was walking down a corridor lost his balance and bumped his head on the edge of a doorway. Because of a previously existing medical condition that made him susceptible to bleeding on the brain, he suffered a cerebral hemorrhage and permanent mental impairment, despite prompt medical attention on the ship. The passenger brought suit against the cruise ship owner for his damages. At trial, the passenger presented evidence of how he was injured as he walked down the hallway, his previous medical condition, and his medical expenses and other damages. The cruise ship owner presented evidence that the cruise ship was following its approved route and that the whale could not have been detected before impact, and that the bump would not have injured someone in ordinary health. At the close of the evidence, the cruise ship owner moved for a directed verdict. How should the court rule? A Grant the motion, because there is no evidence that the crew operated the ship negligently. B Grant the motion, because the cruise ship owner introduced uncontroverted evidence that a person in normal health would not have been injured by the bump. C Deny the motion, because the jury could find that the cruise ship owner, as a common carrier and innkeeper, breached its high duty of care to the passenger. D Deny the motion, because the fact that the severity of the passenger's injuries was not foreseeable does not cut off the cruise ship owner's liability.

The court should grant the cruise ship owner's motion because the passenger has not established a prima facie case of negligence against the cruise ship. To establish a prima facie case for negligence, a plaintiff must show (i) a duty of care, (ii) breach of that duty, (iii) actual and proximate cause, and (iv) damages. As a common carrier and/or an innkeeper, the cruise ship owed its passengers a high duty of care, and therefore would be liable for slight negligence. However, the passenger has offered no evidence to establish that the cruise ship employees breached that duty, and res ipsa loquitur is not applicable here because the collision with the whale swimming underwater is not the type of event that would occur only as a result of negligence. Because the passenger failed to establish breach of duty, the court should grant the cruise ship owner a directed verdict. (B) is incorrect because the cruise ship owner does not need that evidence to prevail. While evidence that a person in normal health would not have been injured by the bump supports the cruise ship's other evidence that it exercised due care, it is not necessary because the passenger has failed to offer evidence that the cruise ship owner breached its duty. On the other hand, if the cruise ship owner had breached its duty of care to its passengers, the fact that a person in normal health would not have been injured by the bump on the head would not be a defense to liability. If a defendant's negligence causes an aggravation of a plaintiff's existing physical illness, the defendant is liable for the damages caused by the aggravation. (C) is incorrect because, as discussed above, the passenger has failed to present evidence that the cruise ship owner breached the high duty of care that it owed to its guests. (D) is incorrect even though it is a true statement of law, as discussed above. The reason the cruise ship owner prevails is because the passenger has failed to establish a prima facie case.

The defendant negligently blocked a road, forcing the plaintiff to take an alternate road that was equally safe. Another driver negligently collided with the plaintiff on that road. The defendant is not liable to the plaintiff because the collision is: A An unforeseeable result caused by a foreseeable intervening force. B An unforeseeable result caused by an unforeseeable intervening force. C A foreseeable result caused by an unforeseeable intervening force. D An independent result caused by a dependent intervening force.

The defendant is not liable because the collision is an unforeseeable result caused by an unforeseeable intervening force. As a general rule, intervening forces that produce unforeseeable results (i.e., results that were not within the increased risk created by defendant's negligence) will be deemed to be unforeseeable and superseding. A superseding force is one that serves to break the causal connection between defendant's initial negligent act and the ultimate injury, and itself becomes a direct, immediate cause of the injury. Thus, the defendant will be relieved of liability for the consequences of his antecedent conduct. Here, the defendant's negligence was an actual cause of the plaintiff's injury because it would not have happened but for the defendant's negligence, but it did not increase the risk that a completely unrelated collision would happen. The result was not caused by a foreseeable intervening force because the other driver's negligence was completely unrelated to any risk created by the original defendant's negligence. Similarly, the collision was not a foreseeable result because traveling on the equivalent alternate road did not make a collision foreseeable.The collision is not an independent result caused by a dependent intervening force because dependent forces are those that are normal responses or reactions to the situation created by the defendant's negligent act, and here the force was unrelated to the defendant's conduct.

A housecleaning agency was given a key to a customer's house so that the agency could have its employees clean while the homeowner was away. After a maid sent by the agency had finished and left the homeowner's house, she went back because she had forgotten her cigarettes. She neglected to lock the door when she left the second time because she was already late for the next job. When the homeowner returned after a few days away, she discovered that her house had been ransacked and several items of jewelry stolen. The front door was open, and there were no signs of forced entry. If the homeowner brings an action against the agency that employed the maid, what is the likely result? A She will not prevail, because she is limited to claims for breach of contract based on her agreement with the agency. B She will not prevail, because the act of the burglar was an independent supervening cause of the homeowner's loss. C She will prevail, because the maid's failure to lock the door created the risk that someone might enter and take the homeowner's valuables. D She will prevail, because when the maid returned after having completed her work, she was technically a trespasser, making the agency vicariously liable for any damage she caused to the premises.

The homeowner will prevail because the maid's negligence increased the risk of criminal conduct by a third party. Criminal acts and intentional torts of third persons are foreseeable independent intervening forces if the defendant's negligence created a foreseeable risk that they would occur. Here, the maid's failure to lock the door was negligent because it created a risk of burglary; hence, the burglary does not cut off the agency's liability for the maid's negligence. As the maid's employer, the agency is vicariously liable under respondeat superior. (A) is wrong because there is nothing in the facts to indicate that the homeowner waived her right to bring tort claims against the agency; having a contractual relationship with a party does not automatically preclude bringing a tort action against the party. (B) is wrong because the burglary was not a superseding cause of the loss; it was within the increased risk caused by the maid's negligence. (D) is wrong because she reentered to retrieve a personal item that she had brought with her when she went to the job; her return just to get the item was within the scope of her employment and would not make her a trespasser.

A new homeowner had two dogs that frequently barked at birds and squirrels in the yard, especially during the day while the homeowner was at work. A neighbor who worked nights was aggravated by the barking, which disturbed his sleep, and decided to let the homeowner know how he felt. One evening, upon learning that the homeowner was entertaining her boss and several clients, the neighbor came to her front door with a boombox and started playing a recording of the dogs barking, putting it at full volume. When the homeowner came to the door, he began yelling at her and berating her in front of her guests for having no consideration for her neighbors, while continuing to play the recording. The homeowner was very upset, especially because her guests decided that they had better leave, and she ended up losing a bonus that her boss was going to give her at the end of the evening. If the homeowner asserts a claim based on intentional infliction of emotional distress against the neighbor, what will be the probable result? A The homeowner will prevail because the neighbor's conduct was extreme and outrageous. B The homeowner will prevail because she suffered pecuniary harm from the neighbor's conduct. C The neighbor will prevail because the homeowner suffered no physical harm. D The neighbor will prevail if the barking from the homeowner's dogs is judged to constitute a nuisance.

The homeowner will probably prevail on a claim for intentional infliction of emotional distress because the neighbor's conduct was sufficiently extreme and outrageous and the other elements of the tort are present. Intentional infliction of emotional distress requires: (i) an act by defendant amounting to extreme and outrageous conduct; (ii) intent to cause severe emotional distress or recklessness as to the effect of defendant's conduct; (iii) causation; and (iv) damages. "Outrageous conduct" is extreme conduct that transcends all bounds of decency. The neighbor's use of the recording and his insults against the homeowner for the benefit of her guests would probably qualify as extreme and outrageous conduct, particularly because there is no evidence that he had previously tried to resolve the problem with the homeowner in a more civilized manner. The neighbor had the requisite intent (either he intended to cause emotional distress or he was reckless as to its effect), there was causation, and the homeowner suffered damages (i.e., she was severely distressed) as a result of the neighbor's actions. (B) is wrong because pecuniary harm is not required for purposes of this tort—all that is required is severe emotional distress. (C) is wrong because, in contrast to negligent infliction of distress, intentional infliction of distress does not require proof of physical harm to recover. (D) is wrong because the fact that the barking constituted a nuisance would not be a defense to conduct amounting to intentional infliction of distress; abatement of a private nuisance by self-help must be preceded by notice to the other party and must be conducted in a reasonable manner.

Which of the following invasion of privacy branches require the plaintiff to show "publicity"? A Intrusion on plaintiff's affairs or seclusion and public disclosure of private facts about plaintiff. B Intrusion on plaintiff's affairs or seclusion and publication of facts placing plaintiff in a false light. C Publication of facts placing plaintiff in a false light and public disclosure of private facts about plaintiff. D Intrusion on plaintiff's affairs or seclusion, publication of facts placing plaintiff in a false light, and public disclosure of private facts about plaintiff.

The invasion of privacy branches based on publication of facts placing the plaintiff in a false light and public disclosure of private facts about the plaintiff require "publicity" concerning the false light or private facts—i.e., widespread dissemination of the facts. Mere publication to a third person is not sufficient for liability. In contrast, invasion of privacy based on intrusion on the plaintiff's affairs or seclusion requires neither publication nor publicity - just the act of intruding.

A columnist for a major metropolitan newspaper had a very antagonistic relationship with the city's mayor. When a restaurant owned by the columnist's family was shut down by city health inspectors, the columnist responded with a column publicizing the shutdown and asserting that it was in retaliation for his prior columns in which he had criticized the mayor. In fact, the mayor had nothing to do with the action by the city health inspectors. While the columnist had no evidence of the mayor's involvement, he believed that there was a connection because "that's how the city works." Can the mayor recover against the columnist for defamation? A No, because the columnist did not act with actual malice. B No, because the columnist had a qualified privilege to explain why he believed his family's business was shut down. C Yes, because the columnist's hostility toward the mayor establishes malice so as to overcome any qualified privilege the columnist had. D Yes, because the columnist should have investigated the accuracy of his claims before publishing the column.

The mayor cannot recover against the columnist because he did not act with actual malice. A public official, such as a mayor, may not recover for defamatory words relating to his official conduct unless there is clear and convincing proof that the statement was made with actual malice, which is defined as knowledge that the statement was false or reckless disregard as to truth or falsity. Reckless conduct is not measured by whether a reasonable person would have investigated before publishing; rather, there must be a showing that the defendant in fact (subjectively) entertained serious doubts as to the truthfulness of his publication. Here, while the columnist had no evidence of the mayor's involvement with the action of the health inspectors, he believed that there was a connection based on his belief as to how the city operates. Hence, he has not acted with actual malice and is not liable to the mayor for defamation. (B) is incorrect because the columnist's qualified privilege applies only to statements made to defend his own actions, property, or reputation. Even if it were to apply to his explanation of why his family's restaurant was shut down, his statements in the column were beyond the scope of the privilege, which does not extend to making a statement to a mass audience whose reading of the statement would not reasonably further his interest in defending himself. Here, the publication in his newspaper column of his explanation as to why the restaurant was shut down was beyond the scope of any privilege he may have had. (C) is incorrect because malice that will result in the loss of a qualified privilege is defined by most courts as knowledge of falsity or reckless disregard as to truth or falsity, rather than hostility or ill-will. As long as the defendant is using a proper occasion for a qualified privilege in a proper way, he will not lose this privilege simply because he bears ill-will toward the plaintiff. (D) is incorrect because the fact that the columnist should have investigated the accuracy of his assertions and did not only establishes negligence on his part. As discussed above, the mayor, as a public official, must show at least reckless disregard as to truth or falsity to recover in a defamation action.

A motorcyclist was injured in a collision and suffered $100,000 worth of injuries, including $20,000 in hospital and physician's bills. The motorcyclist's medical insurance company paid her $20,000 to cover hospital and medical expenses. Later, she filed suit against the driver of the car that struck her motorcycle. When the case came to trial, the jury agreed with the motorcyclist's contention that her injuries were worth $100,000. The jury also determined that the motorcyclist was 30% negligent and that the driver was 70% negligent. How much should the motorcyclist recover from the driver? A $100,000. B $70,000. C $56,000. D $50,000.

The motorcyclist should recover $70,000 from the driver. Under a comparative negligence system, a contributorily negligent plaintiff is allowed to recover a percentage of her damages. The plaintiff's damages are reduced according to her proportionate share of the fault. Thus, the motorcyclist can recover 70% of her total of $100,000 in damages because she was 30% at fault, leaving her with a recovery of $70,000. As a general rule, damages are not reduced or mitigated by reason of benefits received by the plaintiff from other sources, such as health insurance. Therefore, the $20,000 paid by the motorcyclist's insurance company will not reduce the $70,000 in damages to which she is entitled. (A) is incorrect because it fails to reflect the reduction in damages required under comparative negligence. Because the motorcyclist was 30% negligent, she cannot recover the entire $100,000. (C) is incorrect because it is derived from an initial reduction of damages by the amount of the insurance payments ($100,000 minus $20,000, leaving $80,000). This $80,000 figure is then reduced by the 30% negligence of the motorcyclist, leaving an amount of $56,000. However, as noted above, the insurance payments are not allowed to reduce damages. Thus, the 30% reduction is made from the figure of $100,000, not from $80,000. Similarly, (D) is incorrect because it is derived from a reduction of the $70,000 proportionate recovery by the $20,000 insurance payment.

A landowner had a swimming pool and a dressing cabana constructed in her spacious backyard. The pool was entirely within the confines of the landowner's property. However, one corner of the cabana extended a few inches onto a far corner of her neighbor's land. At the time of the construction, neither the neighbor nor the landowner was aware that the cabana extended onto the neighbor's property. Does the neighbor have a cause of action for trespass? A Yes, because the cabana extends onto the neighbor's land. B Yes, because the presence of the cabana on the neighbor's land has caused damage to his property. C No, because the landowner did not actually enter the neighbor's property. D No, because the landowner did not intend to have the cabana encroach on the neighbor's property.

The neighbor will prevail because the cabana extends onto the neighbor's land. The tort of trespass to land requires: (i) an act of physical invasion of the plaintiff's real property by the defendant, (ii) intent by the defendant to bring about a physical invasion of the property, and (iii) causation. The intent required is the intent to enter on a particular piece of land, rather than intent to trespass. Also, it is not necessary that the defendant personally enter the land. It is sufficient if the defendant's act or something set in motion thereby causes a physical invasion of the property. By having the cabana constructed, the landowner acted so as to bring about the physical invasion of the neighbor's land. (C) is incorrect because it makes no difference that the landowner herself did not enter the property that was being violated. (D) is incorrect because the landowner's intent to have the cabana built on its current site suffices for purposes of trespass liability. As noted above, the defendant need not have intended to commit a trespass. (B) is incorrect because actual injury to the violated property is not a prerequisite to sustain this cause of action. Damage is presumed.

A man working at a clothing store discovered that his girlfriend and coworker had been taking money from the cash register. Not wanting to be a party to the situation, he ended the relationship and found another job. Not long after this, the man's new boss, who knew why the man had quit, came into the clothing store. He asked the girlfriend if she missed her boyfriend working with her at the store. She replied, "Yes, but when we found that he was stealing from the cash register, we had no choice but to let him go." If the man sues his former girlfriend for defamation, the fact that the new boss knew the truth of why the man had left his job at the store will have what result? A It will act as a complete defense to an action for defamation. B It will establish that the man has not suffered any actual injury. C It may diminish the damages that the man would be entitled to recover. D It proves that the girlfriend had no reasonable ground for believing that the man was fired for dishonesty.

The new boss's knowledge of the true circumstances behind the man's departure from the store may diminish the man's recovery. The girlfriend is liable for defamation because she made a defamatory statement about the man to a third person. As long as it is understood in its defamatory sense, an accusation need not be believed to be actionable. Because the statement that he was stealing at his job constituted slander per se, damages are presumed. Hence, (A) is wrong. (B) is wrong because actual injury encompasses not only damage to reputation but also humiliation and mental distress, for which the man could recover even though the new boss did not believe the girlfriend's statement. (D) is wrong because the fact that the new boss did not believe the statement does not prove lack of basis for the girlfriend to have made it.

A patient troubled by an irritating skin rash consulted a dermatologist for treatment. The dermatologist diagnosed the rash as a genetic condition that had no cure and would ultimately spread and lead to disfigurement. The patient was shocked and distressed by the diagnosis. On the advice of her family, a week later the patient consulted another doctor. That doctor immediately diagnosed the skin rash as a common bacterial infection and prescribed an ointment that cleared up the condition in a few days. Because the doctor was a friend of the family, the patient was not charged for that visit. Can the patient recover from the dermatologist for the emotional distress caused by his erroneous diagnosis? A Yes, but only if the patient's distress caused her some physical injury. B Yes, because the misdiagnosis by the dermatologist caused the patient actual harm. C No, because the patient did not have to pay for the second doctor visit. D No, because the dermatologist's conduct did not create a foreseeable risk of physical injury to the patient.

The patient's distress is a recoverable element of damages caused by the dermatologist's breach of duty to her. A medical specialist such as the dermatologist owes a duty to possess and exercise the degree of knowledge and skill that dermatologists across the nation exercise. He breached that duty by misdiagnosing a common skin infection that another doctor was able to diagnose immediately. His failure to properly diagnose the condition was the actual and proximate cause of injury to the patient; but for the misdiagnosis, she would not have had to continue suffering from the rash until the other doctor properly treated it. The continuation of the rash and any pain and suffering from it are compensable damages that she can recover from the dermatologist. Also compensable is the emotional distress that she suffered because of the misdiagnosis. While recovery for negligent infliction of emotional distress is not always available in many jurisdictions when there is no other injury caused by the breach, these restrictions do not apply when plaintiff is the victim of another tort that causes physical injury. Plaintiff can recover damages for emotional distress that arise from the tortious conduct. Hence, (A) is incorrect. (C) is incorrect because the patient has suffered compensable injury regardless of whether she had to pay for the second doctor visit. The continuation of the skin rash until she saw the other doctor suffices as the damage element of the prima facie case. (D) is incorrect because, given the patient's physical condition, a failure to make a proper diagnosis did create a foreseeable risk that she would continue to suffer from a painful condition that could otherwise have been alleviated. Thus, the dermatologist's conduct constituted a breach of duty.

Which of the following need NOT be shown by the plaintiff under the attractive nuisance doctrine? A The owner was or should have been aware of the dangerous condition. B The child was lured onto the property by the attractive nuisance. C The condition was likely to cause injury because of the child's inability to appreciate the risk. D The expense of remedying the situation is slight compared with the magnitude of the risk.

The plaintiff does not need to show that the child was lured onto the property by the attractive nuisance. The plaintiff does need to show that the owner was or should have been aware of the dangerous condition, that it was likely to cause injury because of the child's inability to appreciate the risk, and that the expense of eliminating the danger is slight compared with the magnitude of the risk. Under the attractive nuisance doctrine, a landowner has a duty to exercise ordinary care to avoid reasonably foreseeable risk of harm to children caused by artificial conditions on the property. To recover under this doctrine, the plaintiff must show that (i) there is a dangerous condition present on the land of which the owner is or should be aware, (ii) the owner knows or should know that young persons frequent the vicinity of this dangerous condition, (iii) the condition is likely to cause injury, i.e., is dangerous, because of the child's inability to appreciate the risk, and (iv) the expense of remedying the situation is slight compared with the magnitude of the risk.

f a statute providing for a criminal penalty is applicable to a common law negligence case, the statute's specific duty will replace the more general common law duty of care. Which of the following does a plaintiff NOT need to show to prove the availability of the statutory standard? A The plaintiff is in the class intended to be protected by the statute. B The statute was designed to prevent the type of harm that the plaintiff suffered. C The plaintiff suffered physical injury because of the defendant's violation of the statute. D The standards set out in the statute are clearly defined.

The plaintiff need not suffer physical injury from the defendant's violation of the statute. While damages is an element of the prima facie case for negligence, any type of damages, including property damages, will suffice. To prove the availability of the statutory standard, a plaintiff must show that the standards set out in the statute are clearly defined. For the statute to apply, (i) the plaintiff must be in the class intended to be protected by the statute, and (ii) the statute must have been designed to prevent the type of injury that he suffered.

The right of contribution among tortfeasors: A Imposes contribution based on equal shares of the overall liability. B Provides for apportionment of damages in the absence of joint and several liability. C Does not apply against a tortfeasor who is immune from liability. D Applies to intentional torts.

The right of contribution among tortfeasors is a device whereby responsibility is apportioned among those who are at fault. However, it does not apply against a tortfeasor who is immune from liability. If the contributing tortfeasor has a defense that would bar liability, such as intra-family tort immunity, she is not liable for contribution. In most states, contribution is based on relative fault of the various tortfeasors rather than on equal shares of the overall liability. Contribution does not provide for apportionment of damages in the absence of joint and several liability; rather, it can only operate in response to joint and several liability, because it allows any tortfeasor required to pay more than his share of damages under joint and several liability rules to have a claim against the other jointly liable parties for the excess. Contribution does not apply to intentional torts in most states.

A college student borrowed his roommate's notebook computer without permission because he needed to write a term paper that was due the next day. While the computer was sitting open on the student's desk overnight, a water pipe in the ceiling began leaking and water dripped down on the computer, rendering it inoperable. A computer repair service estimated that it would cost $500 to repair all the damaged components. At the time it was damaged, the computer was worth $700. If the roommate sues the student for the damage caused to the computer, what will be the extent of his recovery? A Nothing, because the damage occurred through no fault of the student. B Loss of use damages for the time it was in the student's possession. C $500 in damages. D $700 in damages.

The roommate can recover $700 in damages from the student for conversion. To establish a prima facie case of conversion, the following elements must be proved: (i) an act by the defendant interfering with the plaintiff's right of possession in the chattel that is serious enough in nature or consequence to warrant that the defendant pay the full value of the chattel; (ii) intent to perform the act bringing about the interference with the plaintiff's right of possession, and (iii) causation. Even if the conduct is wholly innocent, liability may attach where the interference is serious in nature. Accordingly, accidentally causing damage to another's chattel may constitute a conversion when the damage occurred while the defendant was using the chattel without permission. Here, the student interfered with the roommate's right of possession in the computer by taking it without permission, and it sustained damages of over 70% of its value while in the student's possession. Hence, the student has committed a conversion. The plaintiff in a conversion case is entitled to damages for the fair market value of the chattel at the time and place of the conversion, which in this case was $700. (A) is incorrect because even though the student was not at fault in the water pipe leaking, the damage occurred while the computer was wrongfully in his possession. (B) is incorrect. Had the computer not been damaged, the roommate's recovery would be limited to loss of use damages under a trespass to chattels theory. However, the serious damage that occurred while the computer was in the wrongful possession of the student warrants a recovery for conversion. (C) is incorrect because the damages remedy for conversion is the fair market value; in effect, there is a forced sale of the item. The student may keep the computer but he is liable to the roommate for the entire value of the computer rather than just the cost of repairs.

Which of the following statements is NOT true under the rule that the tortfeasor takes the victim as he finds him? A The rule applies to the victim's existing physical condition but not his mental condition. B The rule applies in both direct cause cases and indirect cause cases. C The unforeseeable severity of the plaintiff's harm does not relieve the defendant of liability. D The rule is also known as the "eggshell-skull plaintiff" rule.

The rule that the tortfeasor takes the victim as he finds him applies to both the victim's existing physical condition and his mental condition. In both direct cause cases and indirect cause cases, the fact that the extent or severity of the harm was not foreseeable does not relieve defendant of liability; in other words, the unforeseeable severity of the plaintiff's harm, or its extent, is irrelevant under this rule. This rule is also known as the "eggshell-skull plaintiff" rule.

A salesman in a highly competitive field went to the police station to post bond for his son, who had been arrested for possession of a small quantity of narcotics. A photographer for the local newspaper who was at the police station took a picture of the salesman flanked by two bulky police officers. The photo, which looked like the pictures of alleged criminals being taken into custody, ran on a quarter of the front page because it had been a slow news day. The photo was accompanied by a very small caption giving the salesman's name and stating that his son had been arrested for possession of narcotics. The salesman's boss was hypersensitive about the reputation of his company and fired the salesman after he saw the picture in the newspaper. If the salesman brings an invasion of privacy action against the newspaper, what is the most likely basis? A Intrusion upon seclusion B False light publicity C Public disclosure of private facts D Appropriation of plaintiff's picture for commercial purposes

The salesman's basis for an invasion of privacy action will be on the basis that the newspaper published facts about the salesman that placed him in a false light. To establish a prima facie case for invasion of privacy based on publication by defendant of facts placing plaintiff in a false light, the following elements must be proved: (i) publication of facts about plaintiff by defendant placing plaintiff in a false light in the public eye; and (ii) the "false light" must be something that would be highly offensive to a reasonable person under the circumstances. The large picture of the salesman flanked by two bulky police officers could suggest that the salesman committed a crime because it looked like pictures that newspapers often print of alleged criminals being taken into custody.This "false light" would be highly offensive to a reasonable person under the circumstances. (A) is incorrect. This branch of invasion of right to privacy, intrusion upon plaintiff's affairs or seclusion, requires (i) an act of prying or intruding on the affairs or seclusion of plaintiff by defendant; (ii) the intrusion must be something that would be highly offensive to a reasonable person; and (iii) the thing to which there is an intrusion or prying must be "private." Here, the photograph of the salesman was taken at the police station, which is a public place. Hence, the intrusion was not into anything of the salesman's private domain and is not actionable under this branch of invasion of privacy. (C) is similarly incorrect. Public disclosure of private facts requires (i) publication or public disclosure of private information about the plaintiff, and (ii) the matter made public is such that its disclosure would be highly offensive to a reasonable person. Here, the presence of the salesman outside the police station was not a private fact. (D) is incorrect because appropriation of a plaintiff's picture or name for commercial purposes must be for the promotion or advertisement of a product or service; the fact that the defendant is using the picture in a newspaper that it is selling is not sufficient.

To prove breach of duty in a products liability action based on negligence, the plaintiff must show: A Res ipsa loquitur. B The conduct involved was below the level of care generally exercised by the defendant. C The product was dangerous because it departed from its intended design. D The defendant supplied a defective product.

To prove breach of duty in a products liability action, the plaintiff must show (i) negligent conduct by the defendant leading to (ii) the supplying of a defective product by the defendant. Negligent conduct is demonstrated by showing that the defendant's conduct fell below the standard of care expected of a reasonable person under like circumstances, not the level of care generally exercised by the defendant. To show negligence in a manufacturing defect case, the plaintiff may invoke res ipsa loquitur, but it is not required that the plaintiff prove res ipsa loquitur in establishing breach of duty. A plaintiff may show that a product was dangerous because it departed from its intended design to establish a manufacturing defect, but may instead show that the design itself is deficient (to establish a design defect).

A child is required to conform to a higher standard of care than that of a child of like age, education, intelligence, and experience when the child: A Engages in an activity that is dangerous. B Is trespassing at the location of an attractive nuisance. C Has an affirmative duty to act. D Engages in an activity that is normally one in which only adults engage.

When a child engages in an activity that is normally one that only adults engage in, he is required to conform to the same standard of care as an adult in such an activity. The general rule that a child must conform to the standard of care of a child of like age, education, intelligence, and experience does not apply. It is not enough that the child engages in an activity that is dangerous; the activity must be one in which only adults normally engage. When a child is trespassing at the location of an attractive nuisance, the landowner may have a higher standard of care than that owed to adult trespassers, but the child is not required to conform to a higher standard of care. When a child has an affirmative duty to act, he is not required to conform to a higher standard of care than that of a child of like age, education, intelligence, and experience.

Which of the following is correct at common law regarding affirmative duties to act? A One has a general duty to act when it will not result in risk of harm to him B One who acts for the benefit of another has a duty to continue the assistance C A "Good Samaritan" statute excuses any resulting negligence D A physician has a duty to render emergency medical care if it will not result in risk of harm to her

With regard to affirmative duties to act, one who gratuitously acts for the benefit of another is then under a duty to act like an ordinary, prudent, reasonable person and continue the assistance. There is no general duty to act, even if it will not result in risk of harm to the person who would be taking action. As a general matter, no legal duty is imposed on any person to affirmatively act for the benefit of others. Absent a statute changing the common law rule, even physicians are not required to come to the aid of a person needing assistance. A "Good Samaritan" statute does NOT excuse any resulting negligence. Many states have enacted Good Samaritan statutes, which usually exempt doctors, nurses, etc. from liability for ordinary negligence when they voluntarily and gratuitously render emergency treatment. However, liability still exists under most of these statutes for gross negligence.

A golfer and her instructor were playing golf in a foursome when the golfer became very annoyed with critical comments made by the instructor. To show the other golfers in the group how annoyed she was with her instructor, the golfer stood a few yards behind him while the instructor was teeing off and swung a club at him. The instructor, who was focusing on his shot, was not within range of the club but unfortunately the club slipped out of the golfer's hands and struck the instructor in the head, injuring him. If the instructor brings a battery action against the golfer, will he recover? A Yes, because the golfer acted intentionally and caused harmful contact to her instructor. B Yes, because the golfer intended to cause the instructor reasonable apprehension of imminent harmful contact. C No, because the golfer did not intend to cause harmful or offensive contact. D No, unless the golfer acted unreasonably in swinging the club at her instructor.

he golfer will not be liable because she did not intend to cause harmful or offensive contact. The prima facie case for battery has the following elements: (i) an act by the defendant that brings about harmful or offensive contact to the plaintiff's person; (ii) intent on the part of the defendant to bring about harmful or offensive contact to the plaintiff's person; and (iii) causation. Here, the golfer did not have the intent to cause harmful or offensive contact. Hence, she will not be guilty of battery. (A) is incorrect because even though the golfer had the intent to swing the club, she did not have the intent required for battery—to cause harmful or offensive contact to another. (B) is incorrect because the facts do not support an intent to cause an assault. Under the transferred intent doctrine, an intent to cause an assault (intent to cause apprehension of imminent harmful or offensive contact) will satisfy the intent requirement for battery when the other elements of battery are present. Here, however, the golfer was standing behind the instructor and was intending only to show the other golfers how annoyed she was. No intent to commit assault is apparent here. (D) is incorrect because it describes a negligence standard. The instructor may be able to recover against the golfer in a negligence cause of action if the golfer acted unreasonably in swinging the club, but this does not establish intent for a battery action.


Related study sets

Microbi lp 4 Cultivarea bacteriilor

View Set

Basic differences between somatic and automatic nervous system

View Set

TIM 102 Chapter 5 Native Americans

View Set

MGMT 5533 Week 5 Chapts 9 & 10 Authentic and Servant Leadership

View Set

P3 - Abstracting & P4 - Analyzing Problems and Artifacts

View Set

14: Women and Food 2: Focus on Breastfeeding

View Set

Chapter 108: Complementary and Alternative Therapy

View Set